Re: [obm-l] Re: Tangentes trascendentes

2015-08-21 Por tôpico Bernardo Freitas Paulo da Costa
2015-08-21 18:26 GMT-03:00 Israel Meireles Chrisostomo
israelmchrisost...@gmail.com:
 Mas Bernardo e^(pi.i) não é algébrico?Pois e^(pi.i)=-1?

Humpf, eu me apressei botando um pi a mais. A primeira definição que
eu dei, E = exp(i), é que é a certa. E daí não dá exp(pi*i) = -1, como
você falou.

 Em 20 de agosto de 2015 21:46, Bernardo Freitas Paulo da Costa
 bernardo...@gmail.com escreveu:

 2015-08-20 19:50 GMT-03:00 Israel Meireles Chrisostomo
 israelmchrisost...@gmail.com:
  No caso, como provar que tan1 é transcendente?

 tan(1) = sin(1) / cos(1)

 Seja E = exp(i).

 sin(1) = (E - 1/E)/2i
 cos(1) = (E + 1/E)/2

 Logo, se você provar que E = exp(pi*i) é transcendente, você terá
 provado que sin(1) e cos(1) são transcendentes: se eles fossem
 algébricos, E seria raiz de uma equação de segundo grau de um
 algébrico, e portanto também algébrico. Para tan(1) é a mesma coisa:
 se tan(1) fosse algébrico, isso dava uma equação em E, e você veria
 que E é algébrico.

 Mas isso eu só sei fazer usando Lindemann-Weierstrass...
 --
 Bernardo Freitas Paulo da Costa

 --
 Esta mensagem foi verificada pelo sistema de antivírus e
  acredita-se estar livre de perigo.


 =
 Instru�ões para entrar na lista, sair da lista e usar a lista em
 http://www.mat.puc-rio.br/~obmlistas/obm-l.html
 =




 --
 Esta mensagem foi verificada pelo sistema de antivírus e
 acredita-se estar livre de perigo.



-- 
Bernardo Freitas Paulo da Costa

-- 
Esta mensagem foi verificada pelo sistema de antiv�rus e
 acredita-se estar livre de perigo.


=
Instru��es para entrar na lista, sair da lista e usar a lista em
http://www.mat.puc-rio.br/~obmlistas/obm-l.html
=


Re: [obm-l] Re: Tangentes trascendentes

2015-08-20 Por tôpico Bernardo Freitas Paulo da Costa
2015-08-20 19:50 GMT-03:00 Israel Meireles Chrisostomo
israelmchrisost...@gmail.com:
 No caso, como provar que tan1 é transcendente?

tan(1) = sin(1) / cos(1)

Seja E = exp(i).

sin(1) = (E - 1/E)/2i
cos(1) = (E + 1/E)/2

Logo, se você provar que E = exp(pi*i) é transcendente, você terá
provado que sin(1) e cos(1) são transcendentes: se eles fossem
algébricos, E seria raiz de uma equação de segundo grau de um
algébrico, e portanto também algébrico. Para tan(1) é a mesma coisa:
se tan(1) fosse algébrico, isso dava uma equação em E, e você veria
que E é algébrico.

Mas isso eu só sei fazer usando Lindemann-Weierstrass...
-- 
Bernardo Freitas Paulo da Costa

-- 
Esta mensagem foi verificada pelo sistema de antiv�rus e
 acredita-se estar livre de perigo.


=
Instru��es para entrar na lista, sair da lista e usar a lista em
http://www.mat.puc-rio.br/~obmlistas/obm-l.html
=


[obm-l] Re: [obm-l] Re: [obm-l] Determinante máximo

2015-08-19 Por tôpico Bernardo Freitas Paulo da Costa
2015-08-18 23:56 GMT-03:00 Rogerio Ponce abrlw...@gmail.com:
 Ola' Eduardo Henrique,
 imagine o quadrado 4x4 pintado como um tabuleiro de xadrez.
 Para aproveitarmos ao maximo os valores diferentes de zero, eles precisam
 estar todos nas 8 casas de mesma cor.

Faz um certo sentido, mas eu não sei muito bem porquê. Ainda mais,
como o determinante é invariante por permutação de linhas e colunas (a
menos de sinal), se você trocar a segunda linha com a terceira, e a
segunda coluna com a terceira, o determinante é igual mas o padrão
muda.

 Entao o problema se transforma em distribuir estes 8 valores de forma que as
 4 parcelas (diferentes de zero) sejam favoraveis.
 Assim, o maior valor do determinante seria 4, mas precisamos conseguir uma
 arrumacao conveniente.
 Esta daqui, por exemplo, e' suficiente:
 1   0   1   0
 0  -1   0  -1
 1   0  -1   0
 0  -1   0   1

 []'s
 Rogerio Ponce

Eu fiz as contas no computador, e realmente o máximo do determinante é
4. Eu estou pensando num argumento com volumes. O determinante de
uma matriz é sempre menor do que o produto das normas dos vetores
linha (ou coluna) dela. Assim, como temos 8 números diferentes de
zero, faz sentido botar dois em cada linha para que a norma de cada
uma seja raiz(2), e daí o produto é 4. Se não fizermos isso, alguma
linha fica com mais e outra com menos. Como toda linha tem que ter
pelo menos um diferente de zero (senão o det=0), ficam as
possibilidades (para os quadrados)

2,2,2,2 - prod = 16, det = 4
3,2,2,1 - prod = 12, det = 2 raiz(3)
3,3,1,1 - prod = 9, det = 3

Isso mostra que o det = 4, qualquer que seja o caso, e daí basta
achar um caso em que dá certo.

Quanto mais iguais, maior o produto (tem uma desigualdade das médias
escondida?)

Abraços,
-- 
Bernardo Freitas Paulo da Costa

-- 
Esta mensagem foi verificada pelo sistema de antiv�rus e
 acredita-se estar livre de perigo.


=
Instru��es para entrar na lista, sair da lista e usar a lista em
http://www.mat.puc-rio.br/~obmlistas/obm-l.html
=


[obm-l] Re: [obm-l] Re: [obm-l] Função

2015-08-13 Por tôpico Bernardo Freitas Paulo da Costa
2015-08-13 19:38 GMT-03:00 Ralph Teixeira ralp...@gmail.com:
 Tecnicamente, eu diria que f(x)=0 faz o que voce pediu.
E sin(x) ? Mas a pergunta sobre a pergunta é: porquê você quer uma
função assim?

-- 
Bernardo Freitas Paulo da Costa

-- 
Esta mensagem foi verificada pelo sistema de antiv�rus e
 acredita-se estar livre de perigo.


=
Instru��es para entrar na lista, sair da lista e usar a lista em
http://www.mat.puc-rio.br/~obmlistas/obm-l.html
=


[obm-l] Re: [obm-l] Bijeção de intervalos de R com R

2015-08-13 Por tôpico Bernardo Freitas Paulo da Costa
2015-08-13 19:55 GMT-03:00 Israel Meireles Chrisostomo
israelmchrisost...@gmail.com:
 Ralph depois de sua resposta, eu estava pensando e cheguei a uma conclusão
 interessante, talvez eu possa provar que qualquer intervalo de R tem uma
 bijeção com R, usando funções especiais, mas não sei se o meu raciocínio
 está correto.Para isto, preciso recorrer a uma função especial, considere
 uma função especial f(x), cujo domínio seja os reais, se essa função possui
 um mínimo e um máximo, então sua imagem deve estar definida em um intervalo
 fechado, e isto prova que há uma bijeção entre R e um intervalo de R, pois o
 domínio e a imagem devem ser do mesmo tamanho.

Cuidado, você apenas mostrou que tem uma função de R num intervalo.
Você também poderia fazer uma função de R em Z (tipo parte inteira) e
isso não quer dizer que Z e R têm o mesmo tamanho. Ou, pior ainda, o
exemplo f(x) = 0 do Ralph...

 Mas por qual motivo uma
 função que possui um mínimo e um máximo, deve ter sua imagem definida em um
 intervalo fechado?

Falta análise, ou seja, você dizer que a sua função é contínua. Enfim,
eu IMAGINO que você quer dizer que a imagem é o intervalo INTEIRO. (E
não se diz imagem definida em ..., talvez você queira dizer imagem
IGUAL a ...).

 Ralph, este raciocínio prova que há uma bijeção entre R e um intervalo de
 R?Poderia usar a função que vc definiu, cujo domínio pertence aos reais mas
 tem máximo e mínimo...Talvez não prove para todos os intervalos de R...Está
 correto?

Não, você precisaria da função inversa dessa. (o que é um problema
porque a função do Ralph não é bijetiva). Para ajudar a sua intuição,
basta achar duas funções, uma de R sobrejetiva no intervalo (que
mostra que R tem mais do que o intervalo) e uma do intervalo
sobrejetiva em R (que dá a desigualdade no outro sentido). E daí você
usa um canhão (Cantor-Bernstein-Schroder, um argumento mito
bonito) para mostrar que então é igual.

Claro, você pode usar o arcotangente (e variações) para ter funções
explícitas de qualquer intervalo em R, mas em alguns casos você não
vai conseguir coisas bonitinhas como você quer (funções contínuas
com uma só expressão, e tal) porque a topologia vai jogar contra você.
Aliás, essa é uma das partes mais curiosas da topologia, que é dizer
que não adianta, não dá.

Abraços,
-- 
Bernardo Freitas Paulo da Costa

-- 
Esta mensagem foi verificada pelo sistema de antiv�rus e
 acredita-se estar livre de perigo.


=
Instru��es para entrar na lista, sair da lista e usar a lista em
http://www.mat.puc-rio.br/~obmlistas/obm-l.html
=


Re: [obm-l] probleminha

2015-07-31 Por tôpico Bernardo Freitas Paulo da Costa
2015-07-31 12:05 GMT-03:00 Ralph Teixeira ralp...@gmail.com:
 Note:
 S(2n) eh divisivel por 9, entao
 2n eh divisivel por 9, entao
 n eh divisivel por 9, entao
 S(n) eh divisivel por 9, entao
 S(2n) eh divisivel por 81, entao
 S(n) eh divisivel por 144.

 Agora eu vou tentar arrumar algum n que satisfaz esta condicao S(n)=144 e
 S(2n)=81, para pelo menos ter uma ideia do que estah acontecendo... Mas,
 como que S(2n) eh tao menor que S(n), considerando que 2n eh maior que n?
 Ah, os digitos de 2n tem que ser bem menores que os de n...

 Entao vou fazer uma tabela com uma correspondencia entre os digitos de n e
 de 2n:
 Em 1n: 0, 1, 2, 3, 4, 5, 6, 7, 8, 9
 Em 2n: 0, 2, 4, 6, 8, 0, 2, 4, 6, 8 (se nao tiver vai um)
 Em 2n: 1, 3, 5, 7, 9, 1, 3, 5, 7, 9 (se tiver vai um da casa anterior)

 Agora:
 i) Se eu quero S(n)=144 com o menor n, eh essencial colocar poucos
 algarismos -- entao preciso de muitos algarismos grandes. Vou encher n de
 9s...
 ii) Mas ao mesmo tempo eu preciso que S(n)-S(2n)=63, isto eh, eu preciso que
 os digitos de n sejam maiores que os de 2n! Os digitos que melhor
 contribuem para este deficit sao 5 e 6, entao tambem vou encher n de 5
 e 6.
 iii) Minha estrategia de encher n de 5, 6 e 9 significa um monte de
 vai um na soma n+n... Entao vai ter que ser 5 mesmo, que dah o melhor
 deficit quando tem vai um.

Talvez usando 6 em vez de 5 (mas com menos 9's) você chegasse em um
número menor, sei lá, com uma casa decimal a menos. Mas não dá.
Curiosamente, o número n = 699 também satisfaz
S(2n) = 81, mas ele também tem 23 dígitos. Se substituirmos 3 seis por
2 noves no final a soma de 2n vai aumentar (para 90).

 1n=055 555 555 555 555 569 999 999
 2n=111 111 111 111 111 139 999 998

 A boa noticia eh que eu jah garanto que a melhor solucao terah mesmo
 S(n)=144 e S(2n)=81. Afinal, se nao fosse isso, seria S(n)=288, que jah
 seriam 32 algarismos, e meu n ali tem bem menos do que isso.

 Agora precisamos mostrar que esse numero de 23 digitos eh o menor n
 possivel! Ou tem algum menor?

Motivado pelo meu fracasso, eu sei provar que há pelo menos 23 dígitos.
Escreva n = 5 * X + Y, onde X tem apenas zeros e uns, e Y apenas
números de 0 a 4. Assim, 2n = 10X + 2Y, onde não há vai uns na
multiplicação 2Y. Portanto:

S(n) = 5 * S(X) + S(Y)
S(2n) = S(X) + 2*S(Y)

Usando S(n) = 144 e S(2n) = 81 como você já mostrou que basta, temos
que a solução deste sistema é S(X) = 23, ou seja, há 23 vai-uns. O
que quer dizer que toda solução tem que ter pelo menos 23 dígitos.

Mais ainda, a soma dos dígitos em Y é 29 (terminando de resolver o
sistema e o problema!), que devem ser distribuídos o mais para trás
possível no número

55 555 555 555 555 555 555 555

Assim, botamos 7 vezes +4 no final do número, e um +1, que dá a sua solução.


Abraços,
-- 
Bernardo Freitas Paulo da Costa

-- 
Esta mensagem foi verificada pelo sistema de antiv�rus e
 acredita-se estar livre de perigo.


=
Instru��es para entrar na lista, sair da lista e usar a lista em
http://www.mat.puc-rio.br/~obmlistas/obm-l.html
=


[obm-l] Re: [obm-l] Re: [obm-l] Re: [obm-l] Re: [obm-l] transcedência

2015-07-13 Por tôpico Bernardo Freitas Paulo da Costa
2015-07-13 14:44 GMT-03:00 terence thirteen peterdirich...@gmail.com:
 De fato, Teoria de Galois trata exatamente disso. O professor Eduardo Tengan
 deu uma aula sobre isso numa Semana Olímpica:
 www.obm.org.br/opencms/semana_olimpica/docs/2009/galois.ps
 Instale um PS Viewer ou use algum conversor. Enfim, dá para ter uma
 noçãozinha bem legal a partir desse artigo.

 Como não sou matemático formado, eu manjo isso de ler livros por conta mesmo
 :)

 Mas é meio intuitivo: sabe-se que com régua e compasso é possível realizar
 as quatro operações básicas MAIS extração
 de raiz quadrada. Todos os números produzidos dessa forma são raízes de
 polinômios de coeficientes inteiros (mas não quaisquer polinômios, que fique
 claro), e pi é transcedente.

 Logo, pi não pode ser obtido com régua e compasso.

Mais ainda, a mesma teoria de Galois prova que os números
construtíveis com régua e compasso são sempre raízes de polinômios
compostos de quadráticas, porque a única operação não-linear é
intersectar círculos com círculos e retas, que dá coisas de grau 2.
Assim, se um número irracional estiver numa extensão de grau errado
(tipo 3) também não dá para construir. Assim, a Teoria de Galois não
só prova que nenhum transcendente é construtível (e portanto é
impossível a quadratura do círculo com régua e compasso), também a
raíz cúbica de 2 não é construtível (porque o grau da extensão é 3 -
cuidado, nem sempre o grau é tão fácil de calcular como nesse exemplo)
e portanto a duplicação do cubo também não é possível com régua e
compasso.

Abraços,
-- 
Bernardo Freitas Paulo da Costa

-- 
Esta mensagem foi verificada pelo sistema de antiv�rus e
 acredita-se estar livre de perigo.


=
Instru��es para entrar na lista, sair da lista e usar a lista em
http://www.mat.puc-rio.br/~obmlistas/obm-l.html
=


[obm-l] Re: [obm-l] Re: [obm-l] Dízima

2015-06-19 Por tôpico Bernardo Freitas Paulo da Costa
2015-06-19 11:58 GMT-03:00 Esdras Muniz esdrasmunizm...@gmail.com:

 Não é difícil de provar isso, daí vc usa o teorema de Euler pra calcular a 
 ordem: a^φ(n) é congruente a 1 módulo n se mdc(a,n)=1.

É por aí. Primeiro, você tem que mostrar que a ordem de 10 módulo x é
igual ao período de 1/x. Não é difícil, mas tem que mostrar.

Depois, por recorrência, mostre que a ordem de 10 mod 3^n é max(1,
3^(n-2)) (o max taí só para o caso n=1).

Logo o período N será 3^2003. Que tem uma certa quantidade de dígitos
que você calcula com um log.

 Em 19 de junho de 2015 11:55, Esdras Muniz esdrasmunizm...@gmail.com 
 escreveu:

 Cara, acho q é alguma coisa do tipo (ordem de 10 na base 3^2005).Onde a 
 ordem de um número na base b é o menos natural k tal que a^k é congruente a 
 1 módulo b.

 Em 19 de junho de 2015 11:05, Pedro Costa npc1...@gmail.com escreveu:

 Questão do livro( problemas selecionados de matemática - Gandbi- Pág.: 20 
 questão : 63). Já faz dois anos que tento resolver
 este problema e não tem sucesso. Alguém de vocês poderia me ajudar.
 (questão: 63) Seja N o número de algarismos do período da dízima 
 1/(3^2005). O número de algarismos de
 N é igual a:

 a) 952
 b) 953
 c) 954
 d) 955
 e) 956


Abraços,
-- 
Bernardo Freitas Paulo da Costa

-- 
Esta mensagem foi verificada pelo sistema de antiv�rus e
 acredita-se estar livre de perigo.


=
Instru��es para entrar na lista, sair da lista e usar a lista em
http://www.mat.puc-rio.br/~obmlistas/obm-l.html
=


[obm-l] Re: [obm-l] Função periódica

2015-06-11 Por tôpico Bernardo Freitas Paulo da Costa
2015-06-11 8:53 GMT-03:00 marcone augusto araújo borges
marconeborge...@hotmail.com:
 Seja f : R--- R definida por f(x) = sen(ax) + sen(bx), em que a e b são
 constantes reais.

 a) Se a e b são racionais, f é periódica?
Sim.

 b) Vale a recíproca do item anterior?
Não.

 Agradeço por ajuda

Sugiro que você tente mostrar o que acontece quando você soma duas
funções com períodos diferentes, digamos H e L. Depois, tente mostrar
uma condição suficiente para que a soma seja ainda periódica (com,
talvez, outro período).

Abraços,
-- 
Bernardo Freitas Paulo da Costa

-- 
Esta mensagem foi verificada pelo sistema de antiv�rus e
 acredita-se estar livre de perigo.


=
Instru��es para entrar na lista, sair da lista e usar a lista em
http://www.mat.puc-rio.br/~obmlistas/obm-l.html
=


Re: [obm-l] {Filename?} Problema Interessante de Geometria

2015-06-09 Por tôpico Bernardo Freitas Paulo da Costa
2015-06-09 19:54 GMT-03:00 Ralph Teixeira ralp...@gmail.com:
 Oi, Paulo.

 Mas aqui que estah o problema -- nao eh dado que PiPi+1 e igual a QiQi+1,
 soh que sao paralelos... :)

Oi Ralph, Paulo e colegas da lista!

Primeiro, um pedido de clemência: eu não tive muito tempo para pensar
além de ler os emails, e escrever este daqui ;-).

Dado que existe um contra-exemplo para N=4, e que você dá uma condição
a mais para cada novo lado, eu chuto (contando parâmetros, se você
preferir) que sempre haverá contra-exemplos para cada N. O problema é
verificar as condições de fechar o polígono...

Para isso, eu gostaria de pensar no seu problema como uma questão de
descobrir os parâmetros (pontos da reta real) da aplicação de
Schwarz-Christoffel. Os expoentes da fórmula estão fixos (já que eles
determinam os ângulos do problema, que são dados pelo primeiro
polígono). Mas (curiosamente) o problema dá exatamente N dados reais
(as N áreas dos triângulos) e o problema requer N+2 parâmetros (os N
pontos reais e um ponto no interior - que conta por 2 - que
corresponde à origem O). Eu acho que eu estou esquecendo de cancelar
alguma simetria, provavelmente uma translação simultânea de todos os
pontos na direção real. Isso dá N+1 parâmetros.

Espero que eu não tenha adivinhado o problema original ;-)

Abraços,
-- 
Bernardo Freitas Paulo da Costa

-- 
Esta mensagem foi verificada pelo sistema de antiv�rus e
 acredita-se estar livre de perigo.


=
Instru��es para entrar na lista, sair da lista e usar a lista em
http://www.mat.puc-rio.br/~obmlistas/obm-l.html
=


[obm-l] Re: [obm-l] Re: Irredutibilidade de polinômios

2015-05-25 Por tôpico Bernardo Freitas Paulo da Costa
2015-05-24 21:51 GMT-03:00 Gabriel Tostes gtos...@icloud.com:
 Esta certo eu provar isso dizendo que, pelo teorema da raiz racional, as 
 unicas solucoes inteiras podem ser -1, 1, 3 e -3 mas que, com essa opcoes, 
 tal polinomio nunca sera igual a 0?
Não. Pegue dois polinômios irredutíveis em Z[x] sem raízes racionais.
Tipo P = x^2 + 1 e Q = x^2 + 2. O produto deles também não tem raízes
racionais, mas é redutível.

Abraços,
-- 
Bernardo Freitas Paulo da Costa

-- 
Esta mensagem foi verificada pelo sistema de antiv�rus e
 acredita-se estar livre de perigo.


=
Instru��es para entrar na lista, sair da lista e usar a lista em
http://www.mat.puc-rio.br/~obmlistas/obm-l.html
=


[obm-l] Re: [obm-l] Re: [obm-l] Re: [obm-l] Re: Irredutibilidade de polinômios

2015-05-25 Por tôpico Bernardo Freitas Paulo da Costa
2015-05-25 15:30 GMT-03:00 Leonardo Borges Avelino lbor...@gmail.com:
 Daria pra aplicar o critério de Eisenstein diretamente?
Não.

 Dado um polinômio P(x) = a_n*x^n + a_(n-1)*x^(n-1) + ... + a_0 com a_k
 inteiros, suponha que exista um primo p t.q:
 1) p não divide a_n
 2) p divide a_k para k=0, 1, ..., n-1
 3) a_0 não é divisível por p^2
 Então P é irredutível em Q

 Neste problema
 a_n = 1,
 a_(n-1)=5,
 a_(n-2), ..., a_1 =0 e
 a_0=3
 Para um primo p=5, vemos que o o teorema é válido,

p não divide a_0.

 portanto P é irredutível
 em Q. Como mdc dos coeficientes é 1, P também será irredutível em Z

Abraços,
-- 
Bernardo Freitas Paulo da Costa

-- 
Esta mensagem foi verificada pelo sistema de antiv�rus e
 acredita-se estar livre de perigo.


=
Instru��es para entrar na lista, sair da lista e usar a lista em
http://www.mat.puc-rio.br/~obmlistas/obm-l.html
=


[obm-l] Re: [obm-l] Re: [obm-l] Re: [obm-l] Só compartilhando

2015-05-24 Por tôpico Bernardo Freitas Paulo da Costa
2015-05-24 9:13 GMT-03:00 Carlos Nehab carlos.ne...@gmail.com:
 Rsrsrs.
 Vc pensou em dois triângulos equilátero também, cuja razão vale.?
 Possivelmente faltou no enunciado que os triângulos devem ter os três lados
 desiguais.
Não. Eu pensei em ângulos. Tem muitos ângulos interessantes num
triângulo. (já que em geral bastam 3 elementos para fixar o triângulo,
se o problema pede 5 é porque vale informação redundante). E daí
podiam ser os triângulos de lados (5,6,7) e (10,12,14)

E, lendo de novo o enunciado, não tinha ficado claro para mim que era
possível que um elemento (lado, digamos) pudesse corresponder a um
outro (comprimento da bissetriz, p.ex.). Por isso mesmo que eu
perguntei a definição exata de elementos...

 Abs
 Nehab
 Abs

Abraços,
-- 
Bernardo Freitas Paulo da Costa

-- 
Esta mensagem foi verificada pelo sistema de antiv�rus e
 acredita-se estar livre de perigo.


=
Instru��es para entrar na lista, sair da lista e usar a lista em
http://www.mat.puc-rio.br/~obmlistas/obm-l.html
=


[obm-l] Re: [obm-l] Re: [obm-l] Combinatória

2015-05-24 Por tôpico Bernardo Freitas Paulo da Costa
2015-05-24 12:56 GMT-03:00 Rogerio Ponce abrlw...@gmail.com:
 Ola' Marcone,
 os numeros de 9 algarismos comecam em 1, e terminam em 9.
 Indo de um em um, o primeiro e' par, o proximo e' impar, o seguinte e' par,
 etc...
 A sequencia comeca com um par e termina com um impar.
 Portanto tem a mesma quantidade de elementos pares e impares.
 Ou seja, 45000 elementos pares e 45000 elementos impares.

O problema é para a soma dos dígitos ser par ou ímpar. Mas o mesmo
raciocínio funciona, com uma leve mudança: a cada 10, 5 tem a soma dos
dígitos par, e 5, ímpar.

 []'s
 Rogerio Ponce

 2015-05-23 21:31 GMT-03:00 marcone augusto araújo borges
 marconeborge...@hotmail.com:

 Quantos números de 9 algarismos tem a soma dos seus algarismos par?

 Eu achei 45000.Não tenho o gabarito.
 Notei que esse número é a metade do total de números de 9 algarismos
 Seria metade dos números com soma dos seus algarismos par e metade
 com soma dos algarismos ímpar.Se isso for verdade, é mera coincidência
 ou teria como justificar?

Abraços,
-- 
Bernardo Freitas Paulo da Costa

-- 
Esta mensagem foi verificada pelo sistema de antiv�rus e
 acredita-se estar livre de perigo.


=
Instru��es para entrar na lista, sair da lista e usar a lista em
http://www.mat.puc-rio.br/~obmlistas/obm-l.html
=


[obm-l] Re: [obm-l] Re: [obm-l] Combinatória

2015-05-24 Por tôpico Bernardo Freitas Paulo da Costa
2015-05-24 15:38 GMT-03:00 Rogerio Ponce abrlw...@gmail.com:
 A sequencia comeca com um IMPAR e a segunda e' PAR, e vao se alternando
 sucessivamente...

1009 tem soma par
1010 tem soma par também.

Mas a cada 10, 5 são pares, e 5 são ímpares ;-)

 2015-05-24 15:35 GMT-03:00 Rogerio Ponce abrlw...@gmail.com:

 Oi Bernardo, obrigado, engoli a soma.
 Indo de um em um, a soma do primeiro e' par, a proxima e' impar, etc.
 (afinal o Marcone nao queria saber quantos numeros pares existiam na
 sequencia...)

-- 
Bernardo Freitas Paulo da Costa

-- 
Esta mensagem foi verificada pelo sistema de antiv�rus e
 acredita-se estar livre de perigo.


=
Instru��es para entrar na lista, sair da lista e usar a lista em
http://www.mat.puc-rio.br/~obmlistas/obm-l.html
=


[obm-l] Re: [obm-l] Só compartilhando

2015-05-23 Por tôpico Bernardo Freitas Paulo da Costa
2015-05-23 14:55 GMT-03:00 marcone augusto araújo borges
marconeborge...@hotmail.com:
 Determine 2 triângulos não congruentes tais que 5 elementos de um deles
 sejam congruentes a 5 elementos do outro.

O que conta como elementos? Eu tenho uma solução com MUITO mais...
(e que ainda dá lados inteiros)

 Os lados dos triângulos podem se números inteiros?

Abraços,
-- 
Bernardo Freitas Paulo da Costa

-- 
Esta mensagem foi verificada pelo sistema de antiv�rus e
 acredita-se estar livre de perigo.


=
Instru��es para entrar na lista, sair da lista e usar a lista em
http://www.mat.puc-rio.br/~obmlistas/obm-l.html
=


Re: [obm-l] Diofantina

2015-05-20 Por tôpico Bernardo Freitas Paulo da Costa
2015-05-20 13:04 GMT-03:00 Douglas Oliveira de Lima
profdouglaso.del...@gmail.com:

 Hummm acho que consegui!



 Vamos lá, fiz z^2=y^4(assim z deve ser um quadrado perfeito). Logo a equação
 x^2-2z^2=1. Essa equação representa uma hipérbole, então dá pra encontrar as
 soluções racionais nessa curva, fácil ver que (x,y)=(1,0) é solução, assim
 considere uma reta que passa por esse ponto e intercepta a hip. no outro
 ponto (a,b), logo a equação dessa reta será y-0=m(x-1), ou y=m(x-1).

 Agora substituindo na equação da hip. Teremos a seguinte equação do segundo
 grau (1-m^2)x^2+4m^2x-2m^2-1=0, onde conhecemos uma das raízes que é “1”.

 Agora podemos usar o produto para encontrar a outra “a” , assim

 1.a=(-2m^2-1)/(1-2m^2), logo temos todas as soluções racionais da curva

 x^2-2z^2=1, que serão (x,z)=( (-2m^2-1)/(1-2m^2),m((-2m^2-1)/(1-2m^2) -1) ).

 Agora basta verificar para quais valores de m x=(-2m^2-1)/(1-2m^2) será
 inteiro, podemos escrever x=1- 2/(1-2m^2), assim 1-2m^2 deve ser divisor de
 2, ou seja, 1-2m^2=1, 1-2m^2=-1, 1-2m^2=2(não real), 1-2m^2=-2(não inteira),
 onde temos respectivamente os valores m=0, m=-1, m=1 , assim os valores de x
 só podem ser -1 ou 3, e os de z só podem ser 0,2,-2, mas como z deve ser um
 quadrado perfeito só pode ser igual a 0, assim (x,y)=(1,0), ou (x,y)=(-1,0).

m não precisa ser inteiro, pode ser racional. A equação com z, x^2 -
2z^2 = 1 é uma equação de Pell que tem infinitas soluções. O que falta
é mostrar que nenhuma delas tem a segunda coordenada sendo um quadrado
perfeito.

Sol trivial: (x,z) = (1, 0)
Sol fundamental: (x,z) = (3,2)

Outras soluções são obtidas tomando x = parte inteira e y = parte
irracional de (3 - 2*raiz(2))^n. Por exemplo, para n=2 temos
(9 - 2*3*2*raiz(2) + 8) = 17 + 12*raiz(2), e 17^2 = 289 = 2 * 144 + 1.

Abraços,
-- 
Bernardo Freitas Paulo da Costa

-- 
Esta mensagem foi verificada pelo sistema de antiv�rus e
 acredita-se estar livre de perigo.


=
Instru��es para entrar na lista, sair da lista e usar a lista em
http://www.mat.puc-rio.br/~obmlistas/obm-l.html
=


Re: [obm-l] irracionalidade

2015-04-29 Por tôpico Bernardo Freitas Paulo da Costa
Oi,

2015-04-29 15:45 GMT-03:00 Albert Bouskela bousk...@gmail.com:
 Não deve ser essa a proposição, veja:

 (sqrt(3))^3 = 3*sqrt(3) (irracional)
 (sqrt(3+1))^3 = 8 (racional)

O enunciado pede que (sqrt(3) + 1)^3 seja irracional, o que é verdade
nesse caso em particular.

 Enviada em: quarta-feira, 29 de abril de 2015 13:50
 Para: obm-l@mat.puc-rio.br
 Assunto: [obm-l] irracionalidade



 Alguém sabe se é possível provar que:seja k um natural,então se r^k é
 irracional então (r+1)^k também é irracional?

Abraços,
-- 
Bernardo

-- 
Esta mensagem foi verificada pelo sistema de antiv�rus e
 acredita-se estar livre de perigo.


=
Instru��es para entrar na lista, sair da lista e usar a lista em
http://www.mat.puc-rio.br/~obmlistas/obm-l.html
=


Re: [obm-l] exponencial

2015-04-23 Por tôpico Bernardo Freitas Paulo da Costa
2015-04-23 12:10 GMT-03:00 Prof. Vitório Gauss vitorioga...@uol.com.br:
 Nobres,

 Estava tentando resolver

 (×+1)^(2x+1)=2

  X=-3/4

Tem outra solução, também. Algo como 0.443737544568736. Que deve ser
bem irracional...

 ... mas não consegui encontrar a resposta

 Abs

 Vitório

Abraços,
-- 
Bernardo Freitas Paulo da Costa

-- 
Esta mensagem foi verificada pelo sistema de antiv�rus e
 acredita-se estar livre de perigo.


=
Instru��es para entrar na lista, sair da lista e usar a lista em
http://www.mat.puc-rio.br/~obmlistas/obm-l.html
=


[obm-l] Re: [obm-l] A função logarítmica --- um teorema

2015-04-23 Por tôpico Bernardo Freitas Paulo da Costa
2015-04-23 18:10 GMT-03:00 Pedro Chaves brped...@hotmail.com:
 Caros Colegas,

 Alguém poderia demonstrar o teorema baixo?

 TEOREMA - Sendo f (x) uma função real de variável real não identicamente 
 nula, definida e contínua em R+   e tal que   com  quaisquer  x e y   
 positivos f (x . y) = f (x) + f (y) , existe um e um só  a  tal  que f (a) = 
 1 e além disso,  f(x) é o logaritmo de de x na base a.

Oi Pedro,

você está aprendendo análise na reta? Em que livro?

Abraços,
-- 
Bernardo Freitas Paulo da Costa

-- 
Esta mensagem foi verificada pelo sistema de antiv�rus e
 acredita-se estar livre de perigo.


=
Instru��es para entrar na lista, sair da lista e usar a lista em
http://www.mat.puc-rio.br/~obmlistas/obm-l.html
=


[obm-l] Re: [obm-l] Equação diofantina por congruência

2015-04-21 Por tôpico Bernardo Freitas Paulo da Costa
2015-04-21 18:13 GMT-03:00 Pedro Chaves brped...@hotmail.com:
 Caros Colegas,

 Estou tentando resolver por congruência a equação diofantina 13x + 7y = 18, 
 mas não estou conseguindo.
 Só consegui  concluir que 13x é congruente a 4 (mod 7).
 Peço-lhes ajuda.
Coragem:

você tem que inverter 13 mod 7 para continuar a simplificar a
equação. No caso específico é fácil, já que 13 == -1 (mod 7). Assim:

13x == 4 (mod 7), implica que (-1)x == 4 e portanto x == -4 == 3 mod
7. Daí, x = 7k + 3. Substitua na equação original, e corra pro abraço.

 Abraços do Pedro Chaves.


Abraços,
-- 
Bernardo Freitas Paulo da Costa

-- 
Esta mensagem foi verificada pelo sistema de antiv�rus e
 acredita-se estar livre de perigo.


=
Instru��es para entrar na lista, sair da lista e usar a lista em
http://www.mat.puc-rio.br/~obmlistas/obm-l.html
=


Re: [obm-l] Primos consecutivos

2015-04-14 Por tôpico Bernardo Freitas Paulo da Costa
2015-04-14 15:47 GMT-03:00 Pedro Chaves brped...@hotmail.com:
 Caro Pedro José e demais colegas,

 De fato, no meu enunciado eu quis dizer:   a, a+2 e a+4 são primos positivos.

Não, isso de primos negativos é uma generalização que não acrescenta
nada. Em quase todos os lugares que eu conheço os primos são por
definição positivos (exceto a Wikipédia em português, que não cita
fontes confiáveis... segundo ela mesma). Basta ver a Wikipédia em
inglês, francês, alemão, a Wolfram
(http://mathworld.wolfram.com/PrimeNumber.html). Ou, citando um livro,
o José Plínio de Oliveira Santos, Introdução à Teoria dos Números, da
Coleção Matemática Universitária. Porquê? Porque é mais simples assim,
e se quando se generaliza o conceito de primos para outros anéis
aparecem muitas outras noções (por exemplo, o desaparecimento da
fatoração única, ...).

 Nesse  caso, necessariamente a = 3.
 Agora, sem a palavra positivos, serviria realmente também a = -7.
 Obrigado a todos!

Por um Z simples e amigável,
Abraços,
-- 
Bernardo Freitas Paulo da Costa

-- 
Esta mensagem foi verificada pelo sistema de antiv�rus e
 acredita-se estar livre de perigo.


=
Instru��es para entrar na lista, sair da lista e usar a lista em
http://www.mat.puc-rio.br/~obmlistas/obm-l.html
=


[obm-l] Re: [obm-l] RE: [obm-l] RE: [obm-l] Demonstrar por indução(Fibonacci)

2015-04-12 Por tôpico Bernardo Freitas Paulo da Costa
2015-04-12 11:17 GMT-03:00 marcone augusto araújo borges
marconeborge...@hotmail.com:
 F2n = F^2(n+1) - F^2(n-1)
Você precisa reforçar a indução, porque F_(2(n+1)) vai usar F_2n e
F_(2n+1). Daí, você realmente tem que demonstrar não apenas esta
fórmula, mas uma fórmula (semelhante) para F_(2n+1) em função de
outros números de Fibonacci, e mostrar por indução (2n, 2n-1) = (2n,
2n+1) = (2n+2, 2n+1).

Claro que uma das dificuldades do problema vai ser adivinhar qual a
fórmula certa para F_(2n+1). Pensando que deve haver produtos, somas e
diferenças, veja que F_3 = 2 será construído a partir de F_2, F_1 e
F_0 (n = 1, logo sobram n+1, n, n-1). Daí, veja que F_5 = 5 será
construído a partir de F_3, F_2 e F_1 e tente chutar uma fórmula
para ele. Também pode ajudar o fato de você *SABER* qual é a fórmula
para F_2n que será demonstrada, você pode usar isso para tentar
adivinhar os termos que vão aparecer para fazer a recorrência dar
certo.

Abraços,
-- 
Bernardo Freitas Paulo da Costa

-- 
Esta mensagem foi verificada pelo sistema de antiv�rus e
 acredita-se estar livre de perigo.


=
Instru��es para entrar na lista, sair da lista e usar a lista em
http://www.mat.puc-rio.br/~obmlistas/obm-l.html
=


[obm-l] Re: [obm-l] Sequência z_n = (1 + z/n)^n

2015-03-22 Por tôpico Bernardo Freitas Paulo da Costa
2015-03-22 3:37 GMT-03:00 Artur Costa Steiner artur_stei...@yahoo.com:
 Para quais valores do complexo z esta sequência converge? Se convergir para 
 um dado z, o limite tem que ser e^z?

Eu faria à la Euler, com a mesma demonstração que vale para os reais.

Expanda (1 + z/n)^n pela fórmula do binômio, fixando um índice k (que
depois vai para infinito) e majore | Soma_{j  k} C(n,j) (z/n)^j | =
| Soma_{j  k} C(n,j) (|z|/n)^j = erro(z,k). O erro não depende de $n
 k$ (isso é importante, é uma convergência uniforme) e esse é o
pulo de gato do Euler). Uma forma de fazer isso é pedir que os
termos sejam dominados por uma PG de razão r, o que dá a desigualdade

C(n,j+1) * (|z|/n)^(j+1) = r * C(n,j) * (|z|/n)^j para todo j  k =
(n-j) * (|z|/n) = r * j para todo j  k =
|z| = n *r * j /(n-j)   para todo j  k.

Basta ter |z| = r*k para isso ser verdade, já que n/(n-j)  1 (mas
pode ser bem perto, logo a estimativa não é ruim quando fizermos
n-infinito)

E daí temos que a norma do resto é majorada pela soma da PG:

C(n,k+1) * (z/n)^(k+1) / (1 - r) = z^(k+1) / (k+1)! / (1 - r)   (usei
que C(n,k+1) = n^(k+1) / (k+1)!)
que não depende de n, conforme anunciado.

Agora é fácil: faça n - infinito, os termos iniciais (j = k) vão
convergir para a série truncada de e^z, e têm um erro que só depende
de k e z. Enfim, veja que erro(z,k) - 0 quando z está fixo (digamos
|z|  R) e k - infinito.


Obs: combinando esta técnica do Euler com as desigualdades de Cauchy e
um pouco mais de análise complexa, é possível mostrar a seguinte
proposição (ainda não terminei os detalhes, vou tentar enviar em
breve):


Seja f_n(z) uma seqüência de funções holomorfas definidas num domínio
(aberto conexo) Omega, limitadas uniformemente por M neste Omega. Seja
K um compacto contido em Omega, Z um subconjunto de K com pelo menos
um ponto de acumulação. Suponha que f_n(z) - 0 para todo z em Z.
Então f_n - 0 em K.


Muito legal!!

Abraços,
-- 
Bernardo Freitas Paulo da Costa

-- 
Esta mensagem foi verificada pelo sistema de antiv�rus e
 acredita-se estar livre de perigo.


=
Instru��es para entrar na lista, sair da lista e usar a lista em
http://www.mat.puc-rio.br/~obmlistas/obm-l.html
=


[obm-l] Re: [obm-l] Número natural de 100 algarismos.

2015-03-18 Por tôpico Bernardo Freitas Paulo da Costa
2015-03-18 9:00 GMT-03:00 Bernardo Freitas Paulo da Costa
bernardo...@gmail.com:
 2015-03-18 8:18 GMT-03:00 Roger roger@gmail.com:
 Prezados,

 Segue uma questão que há uma semana não consegui uma solução convincente. Se
 alguém puder auxiliar, aguardo, por gentileza.

 1) Seja N um número natural de 100 algarismos, não nulos, que é divisível
 pela soma dos seus algarismos. Um valor possível para a soma dos algarismos
 distintos de N é igual a: a) 10 b) 12 c) 15 d) 16 e) 17

 Uma idéia que eu tive, mas ainda não implementei: números são
 divisíveis por 2^n ou 5^n bastando olhar suas n últimas casas
 decimais. Então, tente fazer a soma dos 100 algarismos ser 128 / 256 /
 512 ou 125 / 625. Espero que dê para achar um múltiplo de um destes
 caras com poucos (e pequenos) algarismos distintos... e depois botar
 um monte de 1.

Com um computador, achei 2122112 = 128 * 16579. Daí, qualquer número,
com o que você bem quiser na frente destes 7 dígitos, é divisível por
128. Se você quiser que a soma dos algarismos distintos seja 3, basta
escolher o número certo de uns e dois para que a soma de todos os
algarismos seja 128. Mais ainda, vários valores de somas possíveis de
um número de algarismos contendo 1 e 2 são possíveis, ou seja 10 = 1 +
2 + 7 (bote um sete, e troque um monte dos 2 por 1. Isso pode
tornar difícil a parte de somas muito grandes (tipo será que existe
um número de 100 dígitos usando todos os 9 algarismos, e que seja
divisível pela soma dos algarismos?) mas daí basta usar o 256.

Estranha questão...
-- 
Bernardo Freitas Paulo da Costa

-- 
Esta mensagem foi verificada pelo sistema de antiv�rus e
 acredita-se estar livre de perigo.


=
Instru��es para entrar na lista, sair da lista e usar a lista em
http://www.mat.puc-rio.br/~obmlistas/obm-l.html
=


[obm-l] Re: [obm-l] Número natural de 100 algarismos.

2015-03-18 Por tôpico Bernardo Freitas Paulo da Costa
2015-03-18 8:18 GMT-03:00 Roger roger@gmail.com:
 Prezados,

 Segue uma questão que há uma semana não consegui uma solução convincente. Se
 alguém puder auxiliar, aguardo, por gentileza.

 1) Seja N um número natural de 100 algarismos, não nulos, que é divisível
 pela soma dos seus algarismos. Um valor possível para a soma dos algarismos
 distintos de N é igual a: a) 10 b) 12 c) 15 d) 16 e) 17

Uma idéia que eu tive, mas ainda não implementei: números são
divisíveis por 2^n ou 5^n bastando olhar suas n últimas casas
decimais. Então, tente fazer a soma dos 100 algarismos ser 128 / 256 /
512 ou 125 / 625. Espero que dê para achar um múltiplo de um destes
caras com poucos (e pequenos) algarismos distintos... e depois botar
um monte de 1.

Provar que as outras somas são impossíveis me parece bem mais difícil...
-- 
Bernardo Freitas Paulo da Costa

-- 
Esta mensagem foi verificada pelo sistema de antiv�rus e
 acredita-se estar livre de perigo.


=
Instru��es para entrar na lista, sair da lista e usar a lista em
http://www.mat.puc-rio.br/~obmlistas/obm-l.html
=


[obm-l] Re: [obm-l] Re: [obm-l] Re: [obm-l] Número natural de 100 algarismos.

2015-03-18 Por tôpico Bernardo Freitas Paulo da Costa
2015-03-18 9:19 GMT-03:00 Roger roger@gmail.com:
 Olá, Bernardo.

 Acredito que seu argumento não é generalizado.

 Contra exemplo: 2^=128
 1.128 não é divisível por 128.
Não é isso. Eu quis dizer que se um número de SETE dígitos for
divisível por 128, então qualquer coisa que você bote na frente
continua divisível. Você só mostrou que qualquer número da forma
abcd0001128 não é divisível por 128.

-- 
Bernardo Freitas Paulo da Costa

-- 
Esta mensagem foi verificada pelo sistema de antiv�rus e
 acredita-se estar livre de perigo.


=
Instru��es para entrar na lista, sair da lista e usar a lista em
http://www.mat.puc-rio.br/~obmlistas/obm-l.html
=


[obm-l] Re: [obm-l] Limite, alguém pode me ajudar?

2015-03-14 Por tôpico Bernardo Freitas Paulo da Costa
2015-03-13 23:47 GMT-03:00 Israel Meireles Chrisostomo
israelmchrisost...@gmail.com:
 Alguém pode me dar uma idéia de como provar que
 lim n →∞ ( x/ncot(x/n)+x/n)^n=e^x

 Estava pensando em usar que  lim n →∞  x/ncot(x/n)=1 e substituir no limite
 obtendo o seguinte:
  lim n →∞ ( x/ncot(x/n)+x/n)^n= lim n →∞ ( 1+x/n)^n=e^x
 Mas não sei se posso fazer isso, pq o limite está dentro da expressão que
 está elevado a n.

Pois é, não pode. Imagine que a_n - 1 quando n - ∞ , que é a única
coisa que você provou.
Nada garante que lim (a_n + x/n)^n convirja, quanto mais para e^x. Os
3 casos clássicos são (sem demonstração, mas é fácil de fazer as
contas):
- a_n = 1 + 1/raiz(n), que vai divergir
- a_n = 1 + (-1)^n/n, que vai oscilar entre e^(x+1) e e^(x-1)
- a_n = 1 + 1/n^2, que vai convergir para e^x

Em termos vagos, tudo depende da velocidade com que a_n tende a 1.

 Se possível, me sugiram uma solução sem usar derivadas(L'Hospital) e por 
 favor,
 me respondam se eu posso fazer isso.

Pois é, não tem como não usar derivadas. Pode ser com L'Hôpital (que
vai dar muitas contas), pode ser de outras formas (expansões em séries
de Taylor truncadas, que aparentemente é um nome que só é conhecido na
França http://fr.wikipedia.org/wiki/Développement_limité), mas você
precisa de mais informação do que apenas o limite dos a_n. Só para
aplicar no seu caso:

x/n * cot(x/n) = y * cot(y) com y - 0 = y / tg(y) = y / (y + O(y^3))
= 1 + O(y^2) = 1 + O( (x/n)^2 )
E daí temos (1 + x/n + O( (x/n)^2 ))^n = exp(x) * (1 + O( (x/n)^2 ))^n - exp(x)

Abraços,
-- 
Bernardo Freitas Paulo da Costa

-- 
Esta mensagem foi verificada pelo sistema de antiv�rus e
 acredita-se estar livre de perigo.


=
Instru��es para entrar na lista, sair da lista e usar a lista em
http://www.mat.puc-rio.br/~obmlistas/obm-l.html
=


Re: [obm-l] Desigualdade

2015-02-18 Por tôpico Bernardo Freitas Paulo da Costa
2015-02-18 11:21 GMT-02:00 Manoel P G Neto Neto buniakov...@yahoo.com.br:
 Caros
 Gostaria de receber uma dica sobre
 a demonstração da desigualdade:

 a^-1+b^-1+c^-1(a^8+b^8+c^8)/a^3b^3c^3
 a, b, c positivos, distintos.

Bunching (também conhecida como Muirhead). Ah, sim, é =, claro (se
a=b=c=1, dá 3 dos dois lados)

 Usei a desigualdade entre as médias, mas não
 consegui.

Abraços,
-- 
Bernardo Freitas Paulo da Costa

-- 
Esta mensagem foi verificada pelo sistema de antiv�rus e
 acredita-se estar livre de perigo.


=
Instru��es para entrar na lista, sair da lista e usar a lista em
http://www.mat.puc-rio.br/~obmlistas/obm-l.html
=


[obm-l] Re: [obm-l] Questão simples

2015-02-09 Por tôpico Bernardo Freitas Paulo da Costa
2015-02-09 0:49 GMT-02:00 marcone augusto araújo borges
marconeborge...@hotmail.com:
 Eu não conheço o project Euler, Bernardo.
 Dei uma olhadinha bem rápida depois da sua citação.
 Essa questão eu formulei porque  um colega mandou uma
 mensagem com uma brincadeira dizendo que ´´às vezes a sorte
 ajuda´´1/4 = 16/64(cancelando os seis).Eu notei que valia para
 outros números como 19/95 e tentei uma explicação.como não
 obtive sucesso eu recorri à lista.
Vou dar a dica básica, que você já pensou no resto: um número ab é
na verdade (10a + b). Agora, você pode cancelar dígitos em 2
posições em cima e 2 embaixo. Cada uma dessas 2*2 = 4
possibilidades dá uma equação com a,b,c,d. Depois, se não me falha a
memória, é na força bruta mesmo...

Abraços,
-- 
Bernardo Freitas Paulo da Costa

-- 
Esta mensagem foi verificada pelo sistema de antiv�rus e
 acredita-se estar livre de perigo.


=
Instru��es para entrar na lista, sair da lista e usar a lista em
http://www.mat.puc-rio.br/~obmlistas/obm-l.html
=


[obm-l] Re: [obm-l] Questão simples

2015-02-08 Por tôpico Bernardo Freitas Paulo da Costa
2015-02-08 21:14 GMT-02:00 Bernardo Freitas Paulo da Costa
bernardo...@gmail.com:
 2015-02-07 14:07 GMT-02:00 marcone augusto araújo borges
 marconeborge...@hotmail.com:
 16/64 = 1/4(´´cancelando´´ 6 com 6´) e 19/95 = 1/5(´´cancelando´´ 9 com 9)
 Quais são os números ab e bc tais que ab/bc = a/c ?
 Essa questão é do Project Euler. Não respondam...

Me apressei, a questão é MUITO POUCO diferente. Marcone: seja mais
preciso com a sua pergunta (e o project Euler já dá uma dica para
você), por exemplo dizendo o que você já tentou fazer nela.

Ou, se é para compartilhar um problema que você achou legal, seja
explícito que este é o caso, pois é muito raro... (não que faltem
problemas legais na lista, mas falta um pouco a iniciativa de pessoas
como o Artur que postavam problemas apenas para que nós
aproveitássemos)

Abraços,
-- 
Bernardo Freitas Paulo da Costa

-- 
Esta mensagem foi verificada pelo sistema de antiv�rus e
 acredita-se estar livre de perigo.


=
Instru��es para entrar na lista, sair da lista e usar a lista em
http://www.mat.puc-rio.br/~obmlistas/obm-l.html
=


[obm-l] Re: [obm-l] Questão simples

2015-02-08 Por tôpico Bernardo Freitas Paulo da Costa
2015-02-07 14:07 GMT-02:00 marcone augusto araújo borges
marconeborge...@hotmail.com:
 16/64 = 1/4(´´cancelando´´ 6 com 6´) e 19/95 = 1/5(´´cancelando´´ 9 com 9)
 Quais são os números ab e bc tais que ab/bc = a/c ?
Essa questão é do Project Euler. Não respondam...

-- 
Bernardo Freitas Paulo da Costa

-- 
Esta mensagem foi verificada pelo sistema de antiv�rus e
 acredita-se estar livre de perigo.


=
Instru��es para entrar na lista, sair da lista e usar a lista em
http://www.mat.puc-rio.br/~obmlistas/obm-l.html
=


[obm-l] Re: [obm-l] Re: [obm-l] Primos em Potências - Uma ajuda

2015-02-03 Por tôpico Bernardo Freitas Paulo da Costa
 Em 24 de janeiro de 2015 08:23, Richard Vilhena ragnarok.liv...@gmail.com
 escreveu:

 Em que condições 10^2n - 10^n  -1 é um  número primo?

 Exemplos: 10^2 -  10- 1 = 89(primo)
  10^4 - 10^2 - 1 = 9899( não é primo)

 Obrigado.

2015-02-03 0:36 GMT-02:00 terence thirteen peterdirich...@gmail.com:
 É bem provável que em poucos valores. Basicamente é saber quando X^2-X-1 é
 primo, X=10^n.

 Mas (X^3+1)/(X+1) não parece ser um bom gerador para tais primos,
Mas X^3 + 1 = (X+1)(X^2 - X + 1). Tem um 2 sobrando nas suas contas.

Para n = 30, o PARI acha que só n = 1,6 e 9 servem.

Abraços,
-- 
Bernardo Freitas Paulo da Costa

-- 
Esta mensagem foi verificada pelo sistema de antiv�rus e
 acredita-se estar livre de perigo.


=
Instru��es para entrar na lista, sair da lista e usar a lista em
http://www.mat.puc-rio.br/~obmlistas/obm-l.html
=


Re: [obm-l] Desigualdade

2015-01-16 Por tôpico Bernardo Freitas Paulo da Costa
2015-01-15 17:32 GMT-02:00 Carlos Yuzo Shine cysh...@yahoo.com:
 Outra maneira, partindo de e^x  1 + x *para todo x  0* (é, aqui parece que
 precisa de pelo menos um pouco de Cálculo),

Não... enfim, precisa de Análise, mas deixando isso de lado:

exp(x) = lim_{n - infinito} (1 + 1/n)^(nx)

Ora, pelo fórmula do binômio (1 + a)^b = 1 + ab + a^2 * Comb(b, 2 a 2)
+ ...  1 + ab
Logo exp(x) = lim (1 + 1/n * nx) = 1 + x

 é

 e^x = (e^(x/2))^2  (1 + x/2)^2 = 1 + x + x^2/4.

 Aqui, aplicamos a desigualdade acima com x/2 no lugar do x.

Fantástico. Isso explica inclusive porque a questão está com um x^2/4
e não x^2/2 (que quem sabe Cálculo poderia usar).

 []'s
 Shine

Aliás, continuando a minha idéia até o segundo termo:

exp(x) = lim (1 + 1/n * nx + (1/n)^2 * (nx * (nx - 1))/2) = lim (1 +
x + x*(x - 1/n)/2) = 1 + x + x^2/2

Então, sei lá qual a razão profunda do x^2/4...
-- 
Bernardo Freitas Paulo da Costa

-- 
Esta mensagem foi verificada pelo sistema de antiv�rus e
 acredita-se estar livre de perigo.


=
Instru��es para entrar na lista, sair da lista e usar a lista em
http://www.mat.puc-rio.br/~obmlistas/obm-l.html
=


Re: [obm-l] Provar que...

2014-12-20 Por tôpico Bernardo Freitas Paulo da Costa
2014-12-20 0:22 GMT-02:00 Maikel Andril Marcelino maikinho0...@hotmail.com:
 Mas 50x51  50², temos um problema!

49*52  50*50 também. Talvez seja melhor cancelar o 50 que aparece dos
dois lados, daí fica 49*51, 48*52, etc, que são (a-b)*(a+b)  a*a. Mas
daí vai sobrar o 100. Falta pouco.

 From: dr.dhe...@outlook.com
 To: obm-l@mat.puc-rio.br
 Subject: RE: [obm-l] Provar que...
 Date: Sat, 20 Dec 2014 05:14:46 +0300


 Tenta reagrupar 100!, talvez algo como (1*100)(2*99)(3*98)...(50*51), dai
 você terá 50 produtos, cada um deles é equiparável a 50² (a saber menor),
 dai tem que argumentar um pouquinho, mas acho que sai.

 Abraços
 Edu

 
 From: maikinho0...@hotmail.com
 To: obm-l@mat.puc-rio.br
 Subject: [obm-l] Provar que...
 Date: Sat, 20 Dec 2014 04:44:26 +0300

 100!  50^100, não estou conseguindo galera. Um abraço Carlos Gomes.

-- 
Bernardo Freitas Paulo da Costa

-- 
Esta mensagem foi verificada pelo sistema de antiv�rus e
 acredita-se estar livre de perigo.


=
Instru��es para entrar na lista, sair da lista e usar a lista em
http://www.mat.puc-rio.br/~obmlistas/obm-l.html
=


Re: [obm-l] Integral

2014-11-27 Por tôpico Bernardo Freitas Paulo da Costa
2014-11-27 13:39 GMT-02:00 João Sousa starterm...@hotmail.com:
 Pessoal, gostaria de uma solução para:

 \int_{-\infty}^{\infty} \frac{x^2}{\sqrt{2\pi \theta}}
 \exp{-\frac{x^2}{2\theta}} dx.

Faça por partes. Dica extra: calcule a derivada de exp(-x^2).
-- 
Bernardo Freitas Paulo da Costa

-- 
Esta mensagem foi verificada pelo sistema de antiv�rus e
 acredita-se estar livre de perigo.


=
Instru��es para entrar na lista, sair da lista e usar a lista em
http://www.mat.puc-rio.br/~obmlistas/obm-l.html
=


[obm-l] Re: [obm-l] Derivadas da função Zeta

2014-11-27 Por tôpico Bernardo Freitas Paulo da Costa
2014-11-27 15:44 GMT-02:00 Amanda Merryl sc...@hotmail.com:
 Oi amigos.

 A função zeta é definida para complexos com Re(z)  1 pela série Z(z) = 
 Soma(k = 1, oo) k^(-z). Embora isto não seja uma série de potências, acho que 
 podemos derivar termo a termo indefinidamente,

Não é uma série de potências, mas é uma série de funções. Reveja os
teoremas de convergência uniforme / normal para séries de funções.
Funções de variável qualquer, claro, para aplicar no caso z
complexo. (Os teoremas que você viu num curso de análise real sobre
séries muitas vezes possuem análogos para mais variáveis, com a mesma
demonstração.)

 de modo, que, se isto for válido, então, no semiplano Re(z)  1, a ngésima 
 derivada é

 Z[n](z) = (-1)^n Soma(k = 2, oo) (ln k)^n k^(-z)

 Eu acho que consegui provar isso no caso de z real. Usando indução e o teste 
 da integral, podemos mostrar que esta série converge para todo real z  1. E 
 como a série primitiva converge, a fórmula acima vale para todo z  1. Mas 
 vale de fato para todo complexo z com Re(z)  1. Se sim, como podemos provar?

Se você estiver fazendo um curso de análise complexa, talvez valha a
pena fazer de outra forma: primeiro, você vai ter que provar que a
zeta é holomorfa. Em seguida, mostrar que a série da derivada n-ésima
também define uma função holomorfa. (Provavelmente, similar à
anterior.) Enfim, conclua que ambas coincidem na reta real, logo por
continuação analítica são iguais em todo { Re(z)  1 }

 Obrigada.

 Amanda

Abraços,
-- 
Bernardo Freitas Paulo da Costa

-- 
Esta mensagem foi verificada pelo sistema de antiv�rus e
 acredita-se estar livre de perigo.


=
Instru��es para entrar na lista, sair da lista e usar a lista em
http://www.mat.puc-rio.br/~obmlistas/obm-l.html
=


Re: [obm-l] Fatorial de inteiro negativo

2014-09-20 Por tôpico Bernardo Freitas Paulo da Costa
2014-09-20 19:23 GMT-03:00 Ennius Lima enn...@bol.com.br:
 Caros Colegas,

 Encontrei um texto de Matemática que define assim o fatorial de um inteiro 
 negativo:

 (-n)! = [(-1)^n].(n!)   (para todo inteiro positivo n)

 Não consegui encontrar, entretanto, outros textos que adotem tal definição. 
 Gostaria de saber o que os Colegas pensam sobre o assunto.
Normal.  A definição mais normal de fatoriais negativos é que eles
sejam infinito, para coincidir com a função Gamma, que tem pólos
nesses pontos. Além disso, esta definição destrói a recorrência: 0! =
0 * (-1)!, logo (-1)! = 1 / 0 = infinito.

Abraços,
-- 
Bernardo Freitas Paulo da Costa

-- 
Esta mensagem foi verificada pelo sistema de antiv�rus e
 acredita-se estar livre de perigo.


=
Instru��es para entrar na lista, sair da lista e usar a lista em
http://www.mat.puc-rio.br/~obmlistas/obm-l.html
=


[obm-l] Re: [obm-l] Área

2014-09-04 Por tôpico Bernardo Freitas Paulo da Costa
2014-09-04 19:00 GMT-03:00 João Sousa starterm...@hotmail.com:
 Considere um sistema cartesiano plano no qual cada unidade no eixo das
 abscissas e no eixo das ordenadas corresponde
 a 1 cm. Sejam os pontos no plano: A(0,0), B(2,0), C(2,3) e D(0,3).


 Ao serem multiplicados pela matriz

 M = 3  1
 4   2

 esses pontos são transformados em E, F, G e H.  Calcule a área,
 em cm2, do quadrilátero cujos vértices são E, F, G e H.
Você já estudou determinantes?

Abraços,
-- 
Bernardo Freitas Paulo da Costa

-- 
Esta mensagem foi verificada pelo sistema de antiv�rus e
 acredita-se estar livre de perigo.


=
Instru��es para entrar na lista, sair da lista e usar a lista em
http://www.mat.puc-rio.br/~obmlistas/obm-l.html
=


[obm-l] Re: [obm-l] Re: [obm-l] Re: [obm-l] Re: [obm-l] Divisíbilidade

2014-08-17 Por tôpico Bernardo Freitas Paulo da Costa
2014-08-16 23:11 GMT-03:00 Gabriel Tostes gtos...@icloud.com:
 xyq=x^2+y^2+1 q=x/y + y/x +1/xy. Como q é inteiro positivo. 1/xy também.

Porque 1/xy tem que ser inteiro positivo? A soma de dois racionais não
inteiros (por exemplo, 5/3 e 1/3) pode ser inteira! E isso não
considera (por exemplo) os casos (já discutidos nessas mensagens) x =
2, y = 5, em que 1/xy = 1/10.

 X=y=1 q=1+1+1=3 alguém mandou essa? Acho q é o jeito mais fácil

Pode acabar chegando na resposta certa, mas certamente o raciocínio está errado.

-- 
Bernardo Freitas Paulo da Costa

-- 
Esta mensagem foi verificada pelo sistema de antiv�rus e
 acredita-se estar livre de perigo.


=
Instru��es para entrar na lista, sair da lista e usar a lista em
http://www.mat.puc-rio.br/~obmlistas/obm-l.html
=


[obm-l] Re: [obm-l] Re: [obm-l] Divisíbilidade

2014-08-15 Por tôpico Bernardo Freitas Paulo da Costa
Eu acho que continua errado...

2014-08-15 11:20 GMT-03:00 Pedro José petroc...@gmail.com:
 x, y Ɛ Z+  e  xy | x^2 + y^2 +1 == x | x^2 + y^2 +1 (i)
 x | x^2 e (i) == x | y^2 + 1 (Combinação Z linear de x^2 + y^2 +1 e x^2)
 == Ǝ k Ɛ Z | kx = y^2 + 1 (ii)
 (ii) e por simetria da proposta ==  Ǝ m Ɛ Z | my = x^2 + 1 == y =( x^2 +
 1)/m (iii)
 (ii) e (iii) == kx = (x^4 + 2x^2 +2)/m^2 == m^2k x = x^4 + 2x^2 +2 (iv)

(ii) kx = y^2 + 1
(iii) y = (x^2 + 1)/m

Donde y^2 = (x^4 + 2x^2 + 1)/m^2
Donde kx = (x^4 + 2x^2 + 1)/m^2 + 1 = (x^4 + 2x^2 + (1 + m^2))/m^2
(e não +2)

O resto talvez funcione mais ou menos igual... mas dá mais trabalho...

 m^2k Ɛ Z (v), pois +, * e potênciação são fechadas em Z.
 (iv) e (v) == x | x^4 + 2x^2 +2 (vi)

 x | x^4 + 2x^2 (vii)
 (vi) e (vii) ==  x | 2 ( Z combinação linear de x^4 + 2x^2 e x^4 + 2x^2 +1)

(k*m*m)*x = (x^4 + 2x^2 + (1 + m*m)) = x | 1 + m^2

Com um pouco de trabalho, você acha também a solução x = 2, y = 5:
10 | 4 + 25 + 1 = 30. E o quociente continua igual a 3 (como
gostaríamos de demonstrar...)

 == x = 1 ou x = 2 e por simetria y=1 ou y=2.
 Pela paradidade da primeira exprexão x ou y Ɛ 2Z + 1. Portanto a solução
 (2,2) não serve.

 x=1 e y= 1 == (x^2 + y^2 + 1)/xy = 3/1 = 3.
 x=1 e y= 2 ==  (x^2 + y^2 + 1)/xy = 6/2 = 3
 x=2 e y=1 ==  (x^2 + y^2 + 1)/xy = 6/2 = 3

 c.q.d.

 nota: O símbolo | significa divide ou tal que a depender do contexto.

 Em vermelho a ocrrência do erro, só fiz y^2 e não y^2 + 1

 Desculpem-me a barbeiragem.

Abraços,
-- 
Bernardo Freitas Paulo da Costa

-- 
Esta mensagem foi verificada pelo sistema de antiv�rus e
 acredita-se estar livre de perigo.


=
Instru��es para entrar na lista, sair da lista e usar a lista em
http://www.mat.puc-rio.br/~obmlistas/obm-l.html
=


[obm-l] Re: [obm-l] Re: [obm-l] Divisíbilidade

2014-08-15 Por tôpico Bernardo Freitas Paulo da Costa
2014-08-15 22:01 GMT-03:00 Bernardo Freitas Paulo da Costa
bernardo...@gmail.com:
 Eu acho que continua errado...

 2014-08-15 11:20 GMT-03:00 Pedro José petroc...@gmail.com:
 x, y Ɛ Z+  e  xy | x^2 + y^2 +1 == x | x^2 + y^2 +1 (i)
 x | x^2 e (i) == x | y^2 + 1 (Combinação Z linear de x^2 + y^2 +1 e x^2)
 == Ǝ k Ɛ Z | kx = y^2 + 1 (ii)
 (ii) e por simetria da proposta ==  Ǝ m Ɛ Z | my = x^2 + 1 == y =( x^2 +
 1)/m (iii)
 (ii) e (iii) == kx = (x^4 + 2x^2 +2)/m^2 == m^2k x = x^4 + 2x^2 +2 (iv)

 (ii) kx = y^2 + 1
 (iii) y = (x^2 + 1)/m

 Donde y^2 = (x^4 + 2x^2 + 1)/m^2
 Donde kx = (x^4 + 2x^2 + 1)/m^2 + 1 = (x^4 + 2x^2 + (1 + m^2))/m^2
 (e não +2)

 O resto talvez funcione mais ou menos igual... mas dá mais trabalho...

 m^2k Ɛ Z (v), pois +, * e potênciação são fechadas em Z.
 (iv) e (v) == x | x^4 + 2x^2 +2 (vi)

 x | x^4 + 2x^2 (vii)
 (vi) e (vii) ==  x | 2 ( Z combinação linear de x^4 + 2x^2 e x^4 + 2x^2 +1)

 (k*m*m)*x = (x^4 + 2x^2 + (1 + m*m)) = x | 1 + m^2

 Com um pouco de trabalho, você acha também a solução x = 2, y = 5:
 10 | 4 + 25 + 1 = 30. E o quociente continua igual a 3 (como
 gostaríamos de demonstrar...)

Bom, com um pouco mais de paciência... você acha a solução x = 5, y =
13. E daí você chuta que a próxima solução é x = 13, y = 34, porque
números de Fibonacci são legais... e dá certo: 34*13 = 442, 13*13 +
34*34 + 1 = 1326 = 3 * 442. Mágica?

Eu acho que há infinitas soluções. Deixo vocês provarem isso. Agora
resta ver que são apenas estas soluções!

Abraços,
-- 
Bernardo Freitas Paulo da Costa

-- 
Esta mensagem foi verificada pelo sistema de antiv�rus e
 acredita-se estar livre de perigo.


=
Instru��es para entrar na lista, sair da lista e usar a lista em
http://www.mat.puc-rio.br/~obmlistas/obm-l.html
=


[obm-l] Re: [obm-l] Sequência de somas inferiores de Riemann convergindo para a integral imprópria

2014-08-11 Por tôpico Bernardo Freitas Paulo da Costa
2014-08-11 14:49 GMT-03:00 Artur Costa Steiner artur_stei...@yahoo.com:

 Boa tarde a todos os amigos. Gostaria de ver a prova de vocês para o seguinte:
Oi Artur,

 Suponhamos que f:(a, b] -- R, a e b reais, seja limitada inferiormente e que 
 sua integral imprópria sobre (a, b] exista e seja finita. Seja (P_n) uma 
 sequência de partições de [a, b] tal que ||P_n|| -- 0. Sendo L_n = L(f, [a, 
 b], P_n) a soma inferior de f sobre [a, b] com relação a P_n, mostre que

 L_n -- Integral (a, b] f(x) dx, integral imprópria.

 Veja que f pode ser ilimitada superiormente. Um bom exemplo é f(x) = 
 1/raiz(x) para x em (0, 1]. Sua integral imprópria é 2.

 Este resultado não é geral. Se vc pegar somas de Riemann arbitrárias, a 
 convergência citada não tem que se verificar. Mesmo que f seja contínua.

Certíssimo. Antes de responder, eu queria saber qual é a sua definição
para integrais impróprias à la Riemann.

-- 
Bernardo Freitas Paulo da Costa

-- 
Esta mensagem foi verificada pelo sistema de antiv�rus e
 acredita-se estar livre de perigo.


=
Instru��es para entrar na lista, sair da lista e usar a lista em
http://www.mat.puc-rio.br/~obmlistas/obm-l.html
=


[obm-l] Re: [obm-l] Dúvida em análise complexa, função Lipschitz

2014-08-10 Por tôpico Bernardo Freitas Paulo da Costa
Lembre que uma função C^1 é localmente Lipschitz.

2014-08-09 16:27 GMT-03:00 Merryl sc...@hotmail.com:
 Eu estou me enrolando nisso.

 Se f é inteira, então f é Lipschitz em todo conjunto limitado do plano 
 complexo. Estou me enrolando para provar, podem ajudar?

 Obrigada
 --
 Esta mensagem foi verificada pelo sistema de antivírus e
  acredita-se estar livre de perigo.


 =
 Instruções para entrar na lista, sair da lista e usar a lista em
 http://www.mat.puc-rio.br/~obmlistas/obm-l.html
 =



-- 
Bernardo Freitas Paulo da Costa

-- 
Esta mensagem foi verificada pelo sistema de antiv�rus e
 acredita-se estar livre de perigo.


=
Instru��es para entrar na lista, sair da lista e usar a lista em
http://www.mat.puc-rio.br/~obmlistas/obm-l.html
=


[obm-l] Re: [obm-l] Raízes irracionais

2014-08-07 Por tôpico Bernardo Freitas Paulo da Costa
2014-08-07 7:21 GMT-03:00 Pedro Chaves brped...@hotmail.com:
 Prezados Colegas,

 Gostaria de saber se alguém conhece uma demonstração do teorema abaixo.
 Um abraço do Pedro Chaves!
 ___

 Teorema das raízes irracionais:


 Seja P(x) um polinômio não identicamente nulo e de coeficientes racionais, e 
 sejam a, m e n números racionais
 — m e n são positivos e m^(1/2) e n^(1/2) são números irracionais. Sejam M = 
 m^(1/2) e N = n^(1/2). Pode-se então afirmar:

 1) Se a + M é raiz de P(x), então a - M também o é (e com a mesma 
 multiplicidade).

 2) Se M + N é raiz de P(x), então M - N, -M + N e -M-N também o são (e todas 
 as quatro raízes têm a mesma multiplicidade).

Essa segunda é falsa. Seja M = raiz(2) e N = 2*raiz(2), de forma que m
= 2 e n = 8. Seja agora P(x) um polinômio com raiz M + N = 3*raiz(2).
Basta tomar P(x) = x^2 - 18. Para este polinômio, -(M+N) também é
raíz, mas nem M-N nem -(M-N) são.

Para ser verdade, você precisa que M e N sejam racionalmente
independentes, o que é (quase) o que você quer mostrar no teorema...

Abraços,
-- 
Bernardo Freitas Paulo da Costa

-- 
Esta mensagem foi verificada pelo sistema de antiv�rus e
 acredita-se estar livre de perigo.


=
Instru��es para entrar na lista, sair da lista e usar a lista em
http://www.mat.puc-rio.br/~obmlistas/obm-l.html
=


Re: [obm-l] Produto de cossenos

2014-08-07 Por tôpico Bernardo Freitas Paulo da Costa
2014-08-07 18:28 GMT-03:00 Walter Tadeu Nogueira da Silveira
wtade...@gmail.com:
 Boa noite a todos.

 Gostaria de uma ajuda.

 Para calcular o produto cos1º.cos2ºcos45º é possível utilizar complexos
 assim: (e^i).(e^2i)...(e^45i) = e^(1+2+...45)i e tomar a parte real?
Não. Veja que nem com apenas dois ângulos de 60° isso dá certo...
cos(60°) = 1/2
cos(60°)^2 = 1/4
Re(e^(60°+60°)i) = Re(e^120°i) = cos(120°) = -1/2

Eu imagino que foi isso que você quis dizer com tomar a parte real,
porque a interpretação literal, ou seja, sem usar os ° para
converter, ia dar errado já para UM ângulo, afinal, cos(45°) = exp (45
* (pi/180) * i) != Re(exp(45i))
 Obrigado

 --
 Walter Tadeu Nogueira da Silveira

Abraços,
-- 
Bernardo Freitas Paulo da Costa

-- 
Esta mensagem foi verificada pelo sistema de antiv�rus e
 acredita-se estar livre de perigo.


=
Instru��es para entrar na lista, sair da lista e usar a lista em
http://www.mat.puc-rio.br/~obmlistas/obm-l.html
=


[obm-l] Re: [obm-l] Re: [obm-l] Re: [obm-l] Quociente da divisão euclidiana

2014-07-14 Por tôpico Bernardo Freitas Paulo da Costa
2014-07-14 17:30 GMT-03:00 Pedro José petroc...@gmail.com:
 Boa tarde!

 É questão de definição.

 Se a e b são inteiros, e b ≠  0, então existem inteiros q e r tais que a =
 bq +r, e 0 ≤ r  |b| .
  Os inteiros q e r, nas condiçõess acima, são únicos. Os inteiros q e r são
 chamados, respectivamente, de
 quociente e resto da divisão euclidiana de a por b.

Hum, não fui claro. Existem pessoas (quem?) que definem a divisão como
você fez, ou seja, com 0 = r  |b|. Neste caso, a pergunta não tem
sentido. Existem outras pessoas (eu, mas principalmente o Arnaldo
Garcia e o Yves Lequain, do livro deles) que definem com 0 = |r| =
|b|. Vale notar que apenas a segunda definição se generaliza
simplesmente para anéis mais gerais. Por exemplo, se você dividir
polinômios, o que entra é 0 = grau(r) = grau(b), e não 0 = r =
grau(b), e é por isso que eu (e, acredito, eles também) preferem a
definição com módulo em r também.

Para dar um outro exemplo de pessoas que não acham que o resto é
positivo: o seu computador também acha que o resto pode ser
negativo... Por exemplo, pergunte quanto dá o resto de (-5) divididos
por (-2). E isso não é só uma linguagem de programação, várias delas
fazem isso, porque a instrução do processador dá resto (-1) para a
pergunta acima. Pro computador, o resto tem o sinal do dividendo.

Talvez fosse questão de fixar (no computador) que o resto é sempre
positivo, mas eu não vejo nenhuma razão para escolher esta
convenção... Uma propriedade curiosa da convenção do computador é
que (-a) % (-b) = -(a % b), o que certamente não vale para a convenção
onde o resto é sempre positivo... Outra propriedade é que (-a) // (-b)
= (a//b) (onde // denota o quociente da divisão com o resto do
computador)

Abraços,
-- 
Bernardo Freitas Paulo da Costa

-- 
Esta mensagem foi verificada pelo sistema de antiv�rus e
 acredita-se estar livre de perigo.


=
Instru��es para entrar na lista, sair da lista e usar a lista em
http://www.mat.puc-rio.br/~obmlistas/obm-l.html
=


[obm-l] Re: [obm-l] Quociente da divisão euclidiana

2014-07-13 Por tôpico Bernardo Freitas Paulo da Costa
2014-07-12 11:53 GMT-03:00 Pedro Chaves brped...@hotmail.com:
 Colegas da lista,

 Sendo m e n inteiros positivos, como provar que o quociente da divisão 
 euclidiana de m por n é maior ou igual a zero?
Ou isso é questão de definição, ou é falso. Para mim, isso é falso: o
quociente da divisão euclidiana é menor do que o divisor, mas
menor é medido pelo valor absoluto, portanto

5 divididos por 3 pode ser 1 com resto 2, ou 2 com resto (-1). Ambas
são igualmente razoáveis, em geral. Aliás, se você dividir em anéis
mais complicados (por exemplo, em Z[i], inteiros de Gauss), pode haver
até quatro restos possíveis!

 Abraços do Pedro Chaves!
-- 
Bernardo Freitas Paulo da Costa

-- 
Esta mensagem foi verificada pelo sistema de antiv�rus e
 acredita-se estar livre de perigo.


=
Instru��es para entrar na lista, sair da lista e usar a lista em
http://www.mat.puc-rio.br/~obmlistas/obm-l.html
=


Re: [obm-l] O mesmo resto

2014-07-10 Por tôpico Bernardo Freitas Paulo da Costa
2014-07-09 21:10 GMT-03:00 Ennius Lima enn...@bol.com.br:
 Olá, pessoal!

 Aproveitando as recentes questões, proponho a demonstração do seguinte 
 teorema:

 O resto da divisão euclidiana de a^m - 1 por a^n - 1 é o mesmo resto da 
 divisão euclidiana de m por n.
 (a, m e n são inteiros positivos; a1 e m=n)

Isso não é um teorema, porque é falso.
Tome a = 2, m = 5, n = 3. 31 mod 7 = 3, 5 mod 3 = 2.

-- 
Bernardo Freitas Paulo da Costa

-- 
Esta mensagem foi verificada pelo sistema de antiv�rus e
 acredita-se estar livre de perigo.


=
Instru��es para entrar na lista, sair da lista e usar a lista em
http://www.mat.puc-rio.br/~obmlistas/obm-l.html
=


Re: [obm-l] Mais uma de diferenciabilidade

2014-07-06 Por tôpico Bernardo Freitas Paulo da Costa
2014-07-05 2:35 GMT+02:00 Merryl sc...@hotmail.com:
 Seja f:I -- R contínua no ponto a do intervalo aberto I. Suponhamos que
 para todas sequências (x_n) e (y_n) em I tais que

 (x_n) seja crescente e convirja para a

 (y_n) seja decrescente e convirja para a

 x_n  a  y_n para todo n

 exista um mesmo real L para o qual convirjam os quocientes ((f(y_n) -
 f(x_n))/(y_n - x_n)).

 Mostre que f é diferenciável em a e que f'(a) = L

Bom, como o Ralph e o Artur já demonstraram, eu não vou escrever a
minha versão, mas apenas comentar para você tentar fazer sozinha sem
copiar as soluções deles, o que é a melhor maneira de aprender.

Essa questão tem vários problemas. O maior é que você tem informação
para o limite de toda sequência e você gostaria de ter uma
informação numa vizinhança. Por assim dizer, um problema de
uniformizar / juntar todos os limites x_n - a numa coisa que é
válida para |x - a|  epsilon. Esse foi o esforço principal do Artur,
que matou o problema no lema que ele mostrou.

O outro problema é que tem variáveis demais, e isso pode atrapalhar.
Note que você pode supor, sem perda de generalidade, que
* a = 0
* f(a) = 0
* L = 0

Nada disso vai resolver o problema, mas pode limpar a área e ajudar
a fazer a parte de cima mais rápido. Isso é justificado com uma
mudança de variáveis (e funções) da seguinte forma: em vez de f(x),
considere a função

g(x) = f(x + a) - f(a) - L*x.

Veja que isso corresponde ao resto da aproximação linear quando você
considera a derivada como a melhor reta que aproxima a sua função.
Aliás, para muitos problemas fundamentais de derivada, é sempre
MUITO poderoso usar a formulação f(x + a) = f(a) + f'(a)*x +
resto(x,a), onde resto(x,a)/x - 0 quando x - 0.

Isso feito, a minha demonstração (que é mais ou menos a do Artur ao
contrário) é assim:

A) Mostre que para y_n - 0 (decrescente) é sempre verdade que
f(y_n)/y_n - 0, usando uma sequência x_n tal que |f(x_n)|  1/2 *
|f(y_n)|, e |x_n|  |y_n| (que existe pela continuidade de f em 0).

B) O passo de uniformizar : suponha por absurdo (como todos fizeram
até aqui, estou ainda tentando achar uma versão direta) que a derivada
não seja L, isso dá uma sequência SPG decrescente (como fez o Ralph)
y_n - 0 tal que f(y_n) é grande. De novo usando o truque  |f(x_n)|
 1/2 * |f(y_n)|, e |x_n|  |y_n| , isso dá um par de sequências x_n
 0  y_n onde o limite não vai dar zero, absurdo.


Abraços,
-- 
Bernardo Freitas Paulo da Costa

-- 
Esta mensagem foi verificada pelo sistema de antiv�rus e
 acredita-se estar livre de perigo.


=
Instru��es para entrar na lista, sair da lista e usar a lista em
http://www.mat.puc-rio.br/~obmlistas/obm-l.html
=


[obm-l] Re: [obm-l] Re: [obm-l] Re: [obm-l] Re: [obm-l] RE: [obm-l] Uma fórmula

2014-06-13 Por tôpico Bernardo Freitas Paulo da Costa
2014-06-13 0:50 GMT-03:00 saulo nilson saulo.nil...@gmail.com:
 1^10 + 2^10 + 3^10 + ... + 100^10
 (0,999+0,001)^10+(1,999+0,001)^10++(99,999+0,001)^10~
 0,999*100+99*100/2+1000*0,001=9,99+1+50*99=4960.99~4961

Impossível... 100^10 = 10^20 que é muito maior do que 4961. Um chute
melhor é (100^11)/10.

-- 
Bernardo Freitas Paulo da Costa

-- 
Esta mensagem foi verificada pelo sistema de antiv�rus e
 acredita-se estar livre de perigo.


=
Instru��es para entrar na lista, sair da lista e usar a lista em
http://www.mat.puc-rio.br/~obmlistas/obm-l.html
=


[obm-l] Re: [obm-l] Re: [obm-l] Re: [obm-l] Máximo e Contagem

2014-06-10 Por tôpico Bernardo Freitas Paulo da Costa
2014-06-10 18:12 GMT-03:00 jamil silva wowels...@gmail.com:
 Ralph, considere  alternado como  =  não consecutivo

HHMMH tem dois ou três H's alternados ? Eu diria que há três H's que
não são consecutivos, mas talvez você queira que contar um H anule
imediatamente os H's vizinhos, A MENOS que haja 4 H's em seguida.

E para ajudar você: ninguém entendeu direito as suas definições,
porque elas são muito pouco precisas. Você sabe responder a pergunta
acima, e fazer exemplos como abaixo, mas a gente não está no seu
cérebro para fazer igual... Eu recomendo que você nos dê um ALGORITMO
para decidir se uma dada sequência é válida ou não. Com isso, ninguém
mais vai te chatear para saber se entendeu certo o que você quer, e é
essa a maior vantagem da matemática, já descoberta por Platão: uma
definição completa é o primeiro passo para o diálogo !

Exemplo de algoritmo:
- Considere primeiro a letra M.
- Se houver 5 M's consecutivos, a disposição é inválida
- Senão, considere todos os grupos de M's consecutivos. Se houver 4
grupos (necessariamente disjuntos), a disposição é inválida

- Repita isso usando a letra H em vez de M
- Se você chegou até aqui, é que a fileira é válida.

- Agora, repita isso em todas as fileiras



 Exemplos:

 H M H M H

 H H M H

 H M H H

 H H M M H

 H H M M M H

 H H H M M M H




 Em 10 de junho de 2014 17:42, Ralph Teixeira ralp...@gmail.com escreveu:

 De novo, você vai ter que dizer explicitamente o que quer dizer por
 alternadas.

 Acho que o significado mais formal de alternada significa UM sim, UM
 não, UM sim, UM não... Mas as pessoas usam esta palavra com outros
 significados -- o mais comum é não necessariamente consecutivas.
 Então HHMH teria 3 homens em posições alternadas -- é assim que você
 quer?

 2014-06-10 16:42 GMT-03:00 jamil silva wowels...@gmail.com:
  Você foi designado para organizar uma sala de reuniões de forma que haja
  em
  cada fileira,
 
  um máximo de quatro pessoas do mesmo sexo em cadeiras vizinhas, ou três,
  em
  cadeiras
 
  alternadas. As fileiras podem ter o mesmo número de pessoas, desde que a
  disposição
 
  entre os sexos seja diferente. Qual o número máximo de pessoas que você
  poderá acomodar,
 
  nestas condições ?
 
 
  --
  Esta mensagem foi verificada pelo sistema de antivírus e
  acredita-se estar livre de perigo.

 --
 Esta mensagem foi verificada pelo sistema de antivírus e
  acredita-se estar livre de perigo.


 =
 Instru�ões para entrar na lista, sair da lista e usar a lista em
 http://www.mat.puc-rio.br/~obmlistas/obm-l.html
 =



 --
 Esta mensagem foi verificada pelo sistema de antivírus e
 acredita-se estar livre de perigo.



-- 
Bernardo Freitas Paulo da Costa

-- 
Esta mensagem foi verificada pelo sistema de antiv�rus e
 acredita-se estar livre de perigo.


=
Instru��es para entrar na lista, sair da lista e usar a lista em
http://www.mat.puc-rio.br/~obmlistas/obm-l.html
=


[obm-l] Re: [obm-l] Re: [obm-l] Re: [obm-l] Par ou Ímpar ? --- Contagem e combinatória

2014-06-08 Por tôpico Bernardo Freitas Paulo da Costa
2014-06-08 15:20 GMT-03:00 Ralph Teixeira ralp...@gmail.com:
 Em geral, em esportes, o pessoal faz a regra com x consecutivas ou
 x+y alternadas; eles costumam usar a palavra alternadas para
 significar apenas em qualquer ordem, nao necessariamente
 consecutivas. Mas aqui fica estranho, nao? O que significa
 alternado neste enunciado? Por exemplo, no caso aaaba voce conta que
 Ana venceu 3 alternadas (digamos, 1a, 3a e 5a) ou nao (apenas dois
 blocos de vitorias)?

 Vou supor que voce estah pensando em blocos de vitorias.
 Consequencia: no caso aaaba, Ana preferiria PERDER a 2a, tornar isso
 ababa, e assim vencer a partida!?!? Eh estranho, porque qualquer regra
 que incentive alguem a fazer gol contra eh estranha para mim... Mas
 vamos lah, vou resolver pensando assim mesmo.

Concordo que é estranho. Entretanto, tem um exemplo do enunciado que
me chamou a atenção:

  Utilize as seguintes convenções:
 
  a = Lance em que Ana acerta
  b = Lance em que Beatriz acerta
  A = Partida em que a vitória é de Ana.
  B = Partida em que a vitória é de Beatriz.
  X = Campeonato de Ana
  Y = Campeonato de Beatriz
 
   = aaba = A
  bbab =  = B

Daqui, eu deduzo que uma partida aaba é suficiente para garantir que
a Ana ganhou. Como não houve 4 vitórias consecutivas, é porque esse
critério se encaixa em 3 vitórias alternadas, que talvez queira
dizer Ana venceu 3 vezes, mas não consecutivas. Assim, o caso aaa
não conta como 3 alternadas porque não houve alternância... Donde:

a) O pior caso para uma partida é ababa (ou babab, ou baabb, ou ...),
em que há 3+2 vitórias+derrotas.
b) O pior caso de um campeonato é ABABABA (ou BABABAB, ou ABBAABA, ou
AAABBBA, ou ...) em que há 4 vitórias+ 3 derrotas.
c) Estou contigo: simetria = 50%.


-- 
Bernardo Freitas Paulo da Costa


 a) A pior hipotese para uma partida seria aaabbbaaabbba, terminando em
 13 lances. De fato, com 13 lances quaisquer, alguem tem que ganhar
 pelo menos 7. Se quem ganhou essas 7 nao o fez em 3 blocos de
 vitorias, entao sao apenas 2 blocos ou menos, e portanto pelo menos um
 bloco terah 4 vitorias consecutivas. Ou seja, em 13, certamente vai
 ter acabado.

 b) Analogamente, o pior caso eh A, com 25
 partidas. De fato, com 25 alguem ganhou pelo menos 13. Se nao ha 4
 blocos (para fazer as 4 vitorias alternadas), entao sao apenas 3;
 como 3x4=1213, pelo menos um dos blocos terah pelo menos 5 partidas
 consecutivas.

 (Em suma, no pior caso seriam 25x13=325 lances de par ou impar. Nao
 parece muito divertido :) :) :) )

 c) Voce diz, as chances de vitoria de Ana versus as de Beatriz? Se
 supusermos que cada **lance** tem probabilidade 50% de vitoria para
 cada uma, por simetria, SIM, ambas tem a mesma chance de ganhar um
 lance, uma partida ou o campeonato.

 Abraco, Ralph.

 Em Sat, 7 Jun 2014 23:01:39 -0300
 jamil silva wowels...@gmail.com escreveu:

  Ana e Beatriz disputam um campeonato de Par ou Ímpar,
  com as seguintes regras:
 
  Uma partida consiste numa série de lances com três acertos
  alternados, ou quatro consecutivos. O campeonato termina quando
  ocorrer quatro vitórias alternadas, ou cinco consecutivas.
 
  Responda:
 
  I) Qual o número máximo de lances de uma partida ?
 
  II) Qual o número máximo de partidas possíveis em um campeonato ?
 
  III) As chances de vitória são iguais ?
 

-- 
Esta mensagem foi verificada pelo sistema de antiv�rus e
 acredita-se estar livre de perigo.


=
Instru��es para entrar na lista, sair da lista e usar a lista em
http://www.mat.puc-rio.br/~obmlistas/obm-l.html
=


Re: [obm-l] perguntinhas simples

2014-06-03 Por tôpico Bernardo Freitas Paulo da Costa
2014-06-02 22:20 GMT-03:00 Hermann ilhadepaqu...@bol.com.br:
 Na 3 vc fez outra questão a minha é Se x+(1/x)^2=3 qual o valor de
 x^3+(1/x)^3?
 não tem quadrado no primeiro x

Bom, na força bruta: x + 1/x^2 = 3 implica que

x^3 + 1 = 3x^2
1 + 1/x^3 = 3/x

Somando as duas igualdades, vem

x^3 + 1/x^3 + 2 = 3x^2 + 3/x = 3x(x + 1/x^2) = 3x * 3

Assim, x^3 + 1/x^3 = 9x - 2, e o valor depende de qual das (três)
raízes do polinômio você vai escolher.

Portanto, eu acho (nessa ordem de plausibilidade) que
- ou tem um quadrado no primeiro x
- ou não tem um quadrado no segundo x
- ou não era uma questão cuja resposta tem um valor numérico
- ou a fórmula com os cubos era mais complicada.

-- 
Bernardo Freitas Paulo da Costa

-- 
Esta mensagem foi verificada pelo sistema de antiv�rus e
 acredita-se estar livre de perigo.


=
Instru��es para entrar na lista, sair da lista e usar a lista em
http://www.mat.puc-rio.br/~obmlistas/obm-l.html
=


[obm-l] Re: [obm-l] Re: [obm-l] Soma trigonométrica

2014-06-03 Por tôpico Bernardo Freitas Paulo da Costa
2014-06-02 17:48 GMT-03:00 Rogerio Ponce abrlw...@gmail.com:
 Ola' pessoal,
 tem um probleminha que se esqueceram de fazer:

Esse problema me parece difícil. Eu só consegui fazer usando raízes da
unidade e polinômios de Chebyshev.

 2014-05-07 8:42 GMT-03:00 Vanderlei Nemitz vanderma...@gmail.com:

 Alguém tem alguma ideia? Tentei utilizar a fórmula da tangente do arco
 duplo, mas ficou complicado.

 Mostre que tg²(1°) + tg²(3°) + tg²(5°) + ...+ tg²(89°)  é um número
 inteiro.


-- 
Bernardo Freitas Paulo da Costa

-- 
Esta mensagem foi verificada pelo sistema de antiv�rus e
 acredita-se estar livre de perigo.


=
Instru��es para entrar na lista, sair da lista e usar a lista em
http://www.mat.puc-rio.br/~obmlistas/obm-l.html
=


Re: [obm-l] Isomorfismo

2014-05-28 Por tôpico Bernardo Freitas Paulo da Costa
2014-05-28 13:19 GMT-03:00 Kleber Santana 1kleb...@gmail.com:
 G={2^m*2^n|m,n Z }, J={m+ni| m,n Z}

 Mostrei que f:J---G é homomorfismo, é injetora, mas não consigo provar que
 é sobrejetora.
Impossível ser injetora. G é de posto 1 enquanto J de posto 2. Talvez
você queira dizer G = {2^m * 3^n } ?
-- 
Bernardo Freitas Paulo da Costa

-- 
Esta mensagem foi verificada pelo sistema de antiv�rus e
 acredita-se estar livre de perigo.


=
Instru��es para entrar na lista, sair da lista e usar a lista em
http://www.mat.puc-rio.br/~obmlistas/obm-l.html
=


Re: [obm-l] Lista 4 Cone Sul 2008

2014-05-21 Por tôpico Bernardo Freitas Paulo da Costa
2014-05-20 8:05 GMT-03:00 Bernardo Freitas Paulo da Costa
bernardo...@gmail.com:
 2014-05-19 23:13 GMT-03:00 terence thirteen peterdirich...@gmail.com:
 Ah, é claro! Uma desigualdade deve resolver!

 n! cresce muito mais rápido que n^2007, então f é estritamente decrescente e
 negativa a partir de certo ponto. Assim ela é certamente injetiva daí, pois
 ab daria f(a)f(b).

 f(a)  f(b), mas é isso.

 O problema agora é antes deste ponto...

 Bom, usando Stirling, n!  n^2007 mais ou menos para n ~ 2007. Mais 
 exatamente,

 n!  n^n / e^n, e esse último é  n^2007 = n^(n - 2007)  e^n = (n
 - 2007) log(n)  n = x * log(x + 2007)  2007 + x
 Como log(2000)  log(729) = log(3^6)  6 log(3)  6, basta 6x  2007 +
 x ou seja x  2007/5 = 401

 Restam 2500 casos para fazer na mão ;-))

Bom, juntando as idéias de divisibilidade e assintótico, sai.

Suponha que 0 = b  a  2500, que é o único caso que resta.

Se b não divide a, mas f(b) = f(a), temos b^2007 - b! = a^2007 - a! =
a! - b! + b^2007 = a^2007. Como b divide a! (pois a  b), isso dá b |
a^2007. Assim, os fatores primos de b aparecem nos fatores primos de
a.

Bom, vamos nos livrar de um caso simples: b = 1. Ou seja, não existe a
 1 tal que a^2007 = a!: Note que (a-1) divide a! (e a-1 diferente de
zero pela nossa hipótese da ordem do a e do b). Mas (a-1) é primo com
a. Logo ele não divide a^2007.

Assim, temos que a = AC e b = BC, onde A e B são primos entre si, C é
um fator comum maior do que 1 (por isso que a gente se livrou do caso
b = 1 antes).

Rearrumando as coisas, temos (AC)! - (BC)! = C^2007 * (A^2007 -
B^2007), logo C^2007 divide (AC)! - (BC)!. Como A  B, temos que C
divide (AC)!/(BC)! = AC * (AC -1) * ... * (AC - BC + 1), logo C não
divide (AC)!/(BC)! - 1. Note que isso vale, na verdade, para qualquer
fator primo (digamos p) de C. Assim, p^2007 divide (AC)! - (BC)! =
(BC)! [ (AC)! / (BC)! - 1 ], logo p^2007 divide (BC)! = b!. Ou seja,
se p é um fator de b, p^2007 divide b!

Mas qualquer primo p aparece menos de b vezes em b!, para qualquer
b: esse número de vezes é igual a

[b/p] + [b/p^2] + ... + [b/p^n] + ...

onde a soma é finita (alguma hora, p^n  b, e as partes inteiras vão
dar zero). Essa soma também é menor do que

b/p * (1/(1-1/p)) (soma da PG sem as partes inteiras) = b / (p - 1) =
b (e só poderia ser igual quando p = 2, e nesse caso, o mais próximo
que chega é (b-1) porque a série é finita, e precisa de termos
fracionários para dar exatamente b/(p-1)).

Agora, note que a  2500, e a = AC, com C  1, implica que a  1250.
Idem para b  a. Assim, p aparece no máximo 1250 vezes em b!, o que é
absurdo pois queríamos que p^2007 | b!


Talvez tenha algo sem usar explicitamente o 2007 e as estimativas de
Stirling, mas saiu !

Abraços
-- 
Bernardo Freitas Paulo da Costa

-- 
Esta mensagem foi verificada pelo sistema de antiv�rus e
 acredita-se estar livre de perigo.


=
Instru��es para entrar na lista, sair da lista e usar a lista em
http://www.mat.puc-rio.br/~obmlistas/obm-l.html
=


Re: [obm-l] Lista 4 Cone Sul 2008

2014-05-21 Por tôpico Bernardo Freitas Paulo da Costa
2014-05-21 13:11 GMT-03:00 terence thirteen peterdirich...@gmail.com:
 Só tem que lembrar que Stirlinbg é pesado demais pra galera. Talvez uma
 desigualdade mais bobinha que saia com indução...

Bom, a demonstração funciona com b  2007, ou seja, podemos ter até a
= 2*2007, ou seja, tem que mostrar que f(n) = n^2007 - n! é
decrescente e negativa para n  4000. Vou deixar você achar a
desigualdade por indução ;-)
-- 
Bernardo Freitas Paulo da Costa

-- 
Esta mensagem foi verificada pelo sistema de antiv�rus e
 acredita-se estar livre de perigo.


=
Instru��es para entrar na lista, sair da lista e usar a lista em
http://www.mat.puc-rio.br/~obmlistas/obm-l.html
=


Re: [obm-l] Lista 4 Cone Sul 2008

2014-05-20 Por tôpico Bernardo Freitas Paulo da Costa
2014-05-19 23:13 GMT-03:00 terence thirteen peterdirich...@gmail.com:
 Ah, é claro! Uma desigualdade deve resolver!

 n! cresce muito mais rápido que n^2007, então f é estritamente decrescente e
 negativa a partir de certo ponto. Assim ela é certamente injetiva daí, pois
 ab daria f(a)f(b).

f(a)  f(b), mas é isso.

 O problema agora é antes deste ponto...

Bom, usando Stirling, n!  n^2007 mais ou menos para n ~ 2007. Mais exatamente,

n!  n^n / e^n, e esse último é  n^2007 = n^(n - 2007)  e^n = (n
- 2007) log(n)  n = x * log(x + 2007)  2007 + x
Como log(2000)  log(729) = log(3^6)  6 log(3)  6, basta 6x  2007 +
x ou seja x  2007/5 = 401

Restam 2500 casos para fazer na mão ;-))
-- 
Bernardo Freitas Paulo da Costa

-- 
Esta mensagem foi verificada pelo sistema de antiv�rus e
 acredita-se estar livre de perigo.


=
Instru��es para entrar na lista, sair da lista e usar a lista em
http://www.mat.puc-rio.br/~obmlistas/obm-l.html
=


Re: [obm-l] Lista 4 Cone Sul 2008

2014-05-20 Por tôpico Bernardo Freitas Paulo da Costa
2014-05-20 12:21 GMT-03:00 terence thirteen peterdirich...@gmail.com:
 MAS acho que podemos melhorar:

 Minha impressão é que a sequência é negativa a partir de certo ponto, e
 antes disso é positiva. Mas em cada trecho ela deve ser monótona. Como
 negativos não são iguais a positivos (exceto na França em que 0 é positivo E
 negativo :P), o problema acabaria.

Não, a sequência (n^2007 - n!) vale zero em n = 1 (e -1 em n = 0).
Para n = 2, isso já vale 2^2007 - 2 = número grandão. E vai AUMENTANDO
até chegar no ponto em que a derivada é zero. E só depois diminui. O
problema, portanto, é ver que essa parábola não passa duas vezes no
mesmo ponto com coordenadas inteiras...
-- 
Bernardo Freitas Paulo da Costa

-- 
Esta mensagem foi verificada pelo sistema de antiv�rus e
 acredita-se estar livre de perigo.


=
Instru��es para entrar na lista, sair da lista e usar a lista em
http://www.mat.puc-rio.br/~obmlistas/obm-l.html
=


[obm-l] Re: [obm-l] Sistema não linear

2014-05-06 Por tôpico Bernardo Freitas Paulo da Costa
2014-05-05 22:04 GMT-03:00 Vanderlei Nemitz vanderma...@gmail.com:
 Como determinar as soluções reais do seguinte sistema?

 x^3 - 3x = y
 y^3 - 3y = z
 z^3 - 3z = x

Por substituição. A primeira dá y em função de x, a segunda dá z em
função de y (logo de x), o que dá uma equação de grau 27 (se não errei
as contas) para x. Ache as 27 soluções e, se x for real, as equações
acima dão que y e z também serão reais.

Moleza! (Se você tem um computador ou o Wolfram Alpha). Senão, você
pode tentar o critério de raízes racionais para ver se tem alguma
raíz fácil.

Nesse caso particular, você pode usar a simetria do problema para
ajudar. Veja que, se x = y, y = x^3 - 3x = y^3 - 3y = z, ou seja os
três são iguais. Daí, você tem que resolver uma equação simples

x^3 = 4x = x = 0 ou x^2 = 4 = x = 0, -2, 2. Isso dá três soluções.

Agora, considere a função f(t) = t^3 - 4t, que é crescente para t  2.
Se x  2, y = x + f(x)  x + f(2)  x. Daí, z = y + f(y)  y + f(2) 
y. Enfim, x = z + f(z)  z + f(2)  z  y  x. Absurdo. A mesma coisa
vale para x  -2. Daí, basta ver se há raízes para -2  x  2, além de
x = 0. Eu fiz uns esboços do gráfico de g(t) = t^3 - 3t, parece que há
outras soluções, mas não sei como calcular sem usar o polinômio de 27o
grau.

Abraços,
-- 
Bernardo Freitas Paulo da Costa

-- 
Esta mensagem foi verificada pelo sistema de antiv�rus e
 acredita-se estar livre de perigo.


=
Instru��es para entrar na lista, sair da lista e usar a lista em
http://www.mat.puc-rio.br/~obmlistas/obm-l.html
=


[obm-l] Re: [obm-l] Re: [obm-l] Re: [obm-l] Matemágica - Séries

2014-04-28 Por tôpico Bernardo Freitas Paulo da Costa
2014-04-28 19:20 GMT-03:00 luiz silva luizfelipec...@yahoo.com.br:
 Pois é,

 Mas a minha pergunta é : se os axiomas não se aplicam, pq quando usados,
 chegam à mesma resposta que os outros métodos corretos (o metodo que vc
 colocou, o uso da função zeta, para o caso dos numeros naturais, etc..)?

Para completar a resposta do Ralph: por sorte. Uma forma de ver essas
manipulações é como se fosse um limite de PGs (e outras coisas
parecidas). Quando você bota 1 (ou -1) numa PG, ela não tem soma.
Mas, em alguns casos, o limite faz sentido. Se você fizer as operações
certas (o que, sem escrever as PGs subjacentes é impossível acertar
sem ter MUITA sorte), você acaba calculando os limites certos, porque
os infinitos (e outras bizarrices) se cancelam mutuamente. Eu lembro
quando me mostraram os vídeos, e eu tentei justificar as contas... as
primeiras tentativas de escrever PGs óbvias para calcular os limites
deram MUITO errado. Claro, usando a zeta é mais fácil de saber a
resposta - se você sabe calcular, e só funciona neste caso particular.

 Em Segunda-feira, 28 de Abril de 2014 17:36, Ralph Teixeira
 ralp...@gmail.com escreveu:
 Só tem um problema -- os axiomas básicos da aritmética dos números naturais
 NÃO se aplicam a somas infinitas. Na definição mais básica de soma infinita,
 a soma 1-1+1-1+1... simplesmente não existe (a série diverge). A soma
 infinita não é associativa, então o que eles fizeram não está nem um pouco
 certo.

 Note-se que HÁ maneiras de REDEFINIR somas infinitas que dão a resposta 1/2
 (somar a Cesaro, por exemplo, vide
 http://en.wikipedia.org/wiki/Ces%C3%A0ro_summation)

 (Só para ilustrar -- um dos axiomas básicos afirma que a soma de números
 positivos dá positivo, o que não é o caso ali.)

 Não é que o que está sendo dito ali está **errado**, mas eu particularmente
 não gosto do jeito que eles apresentam as coisas, fazendo tudo parecer fácil
 e simples quando não é. No fundo no fundo, eles sabem que estão provocando
 (porque os comentários do Youtube são a fonte natural de comentários
 lógicos Viu a ironia?).

Bom, eu sou mais chato do que você. O que estes vídeos fizeram está
matematicamente errado. E o tom do vídeo me deixa muito chateado,
porque incita nas pessoas a idéia que matemática é coisa de maluco /
coisa sem sentido e, pior ainda, matemática é só sair fazendo conta
sem entender o significado. Seria muito melhor que, em vez de ficar
fazendo contas malucas e sem sentido ele explicasse que 1/ As
manipulações estão sendo feita COM OUTRAS CONVENÇÕES 2/ Que existem
diversas outras convenções 3/ Que todas elas coincidem (quando a
resposta existe) com continuação analítica 4/ E que é por isso que a
Física moderna usa muita análise complexa, para poder fazer todas
essas manipulações COM ALGUM SENTIDO. Mas, como disse o Ralph, postar
isso no Youtube dessa forma é muito limitado.

Se alguém quiser entender isso de verdade, eu sugiro começar com um
post do Terence Tao:
http://terrytao.wordpress.com/2010/04/10/the-euler-maclaurin-formula-bernoulli-numbers-the-zeta-function-and-real-variable-analytic-continuation/

Abraços,
-- 
Bernardo Freitas Paulo da Costa

-- 
Esta mensagem foi verificada pelo sistema de antiv�rus e
 acredita-se estar livre de perigo.


=
Instru��es para entrar na lista, sair da lista e usar a lista em
http://www.mat.puc-rio.br/~obmlistas/obm-l.html
=


[obm-l] Re: [obm-l] n(n+1)(n+2)... (n+p-1) é múltiplo do fatorial de p

2014-04-12 Por tôpico Bernardo Freitas Paulo da Costa
2014-04-12 7:21 GMT-03:00 Ennius Lima enn...@bol.com.br:
 O objetivo é fazer a demonstração, ignorando resultados da Análise 
 Combinatória.

Então, a saída é usar indução. Duas variáveis inteiras (p, n) implicam
duas induções. Nesse caso, dá para separar.

Depois, faça indução em p e em seguida em n. A base em p, p = 1 é
fácil. A base em n, n = 1 dá p! / p! também. O passo de indução em n
troca n por (n+p). O termo do meio é divisível por (p-1)! (hipótese
da indução em p-1) logo basta ver o que acontece com os fatores de p.
Como o produto anterior era divisível por p!, os únicos fatores que
faltam estão no n que você retirou, e são fatores de p. Mas esses
fatores reaparecem em (n+p) com (pelo menos) a mesma multiplicidade.

Abraços,
-- 
Bernardo Freitas Paulo da Costa

-- 
Esta mensagem foi verificada pelo sistema de antivírus e
 acredita-se estar livre de perigo.


=
Instruções para entrar na lista, sair da lista e usar a lista em
http://www.mat.puc-rio.br/~obmlistas/obm-l.html
=


[obm-l] Re: [obm-l] Preciso de uma ajuda para a solução de um problema.

2014-03-12 Por tôpico Bernardo Freitas Paulo da Costa
2014-03-11 23:11 GMT-03:00  douglas.olive...@grupoolimpo.com.br:
 Olá , boa noite a todos os amigos da lista, recebi recentemente um problema
 abaixo.

 Determinar a soma de todos os valores de x tais que
 ((x^2+1000x)^(1/2)-x)^(1/2) seja inteiro.

x é inteiro?

-- 
Bernardo Freitas Paulo da Costa

-- 
Esta mensagem foi verificada pelo sistema de antivírus e
 acredita-se estar livre de perigo.


=
Instruções para entrar na lista, sair da lista e usar a lista em
http://www.mat.puc-rio.br/~obmlistas/obm-l.html
=


[obm-l] Re: [obm-l] Quadrado e quadrilátero

2014-03-09 Por tôpico Bernardo Freitas Paulo da Costa
2014-03-09 9:53 GMT-03:00 marcone augusto araújo borges
marconeborge...@hotmail.com:
 Os vértices de um quadrilátero estão sobre os lados de um quadrado cujo lado
 mede 1.Se as medidas dos lados do quadrilátero são a,b,c e d,mostre que
 2  = a^2+b^2+c^2+d^2  = 4

 O quadrado fica repartido em quatro triângulos retângulos e o quadrilátero
 citado
 Sejam x,(1-x),y,(1-y),z,(1-z),w,(1-w) as medidas dos os catetos desses
 triângulos.Temos que a media aritmetica das
 medidas desses catetos é 1/2.Usando MA  = MQ temos que:
 1/2  = [ (a^2+b^2+c^2+d^2)/8 ]^1/2 = (aqui eu agrupei dois a dois e usei o
 teorema de Pitágoras)
 1/4   =  (a^2+b^2+c^2+d^2)/8 =
 2  = a^2+b^2+c^2+d^2
 A outra parte da questão tento e não consigo

Você mostrou que a^2 + ... + d^2 = x^2 + ... + w^2 + (1-x)^2 + ... + (1-w)^2.

Agora estude a função t^2 + (1-t)^2. O mínimo dela é 1/2 (em t = 1/2,
como você demonstrou usando MA = MQ) e o máximo dela é 1 (em t = 0 ou
em t = 1). Isso mostra as duas desigualdades.

Abraços,
-- 
Bernardo Freitas Paulo da Costa

-- 
Esta mensagem foi verificada pelo sistema de antivírus e
 acredita-se estar livre de perigo.


=
Instruções para entrar na lista, sair da lista e usar a lista em
http://www.mat.puc-rio.br/~obmlistas/obm-l.html
=


[obm-l] Re: [obm-l] Polinômio

2014-03-08 Por tôpico Bernardo Freitas Paulo da Costa
2014-03-07 12:57 GMT-03:00 Carlos Nehab carlos.ne...@gmail.com:
 Faça p(x) : (x-1)(x-2)(x-3)Q(x) mais ax2 mais bx mais c e aplique as três
 condições.
 Nehab

Isso dá três equações lineares para a, b, c, o que permite
determiná-los. Eu duvido que eles sejam inteiros, mas eles certamente
serão racionais. Porque isso seria incompatível com p(x) ter
coeficientes inteiros ? Não seria possível que Q(x) também tivesse
coeficientes racionais e com isso cancelasse magicamente os
racionais que porventura aparecessem em a,b e/ou c ?

Abraços,
-- 
Bernardo Freitas Paulo da Costa

-- 
Esta mensagem foi verificada pelo sistema de antivírus e
 acredita-se estar livre de perigo.


=
Instruções para entrar na lista, sair da lista e usar a lista em
http://www.mat.puc-rio.br/~obmlistas/obm-l.html
=


[obm-l] Re: [obm-l] Re: [obm-l] Polinômio

2014-03-08 Por tôpico Bernardo Freitas Paulo da Costa
2014-03-08 14:41 GMT-03:00 Cláudio Gustavo claudiog...@yahoo.com.br:
 Num polinômio com coeficientes inteiros, ao se substituírem dois valores 
 quaisquer a e b do domínio e subtraindo as expressões de p(b) e p(a) eh 
 possível colocar o fator b-a em evidencia. Observando que o outro fator que 
 multiplica b-a continua sendo inteiro, tem-se que (p(b)-p(a))/(b-a) eh 
 inteiro e que b-a divide p(b)-p(a).

Eu não contestei a sua solução, Cláudio. O meu problema é com a
solução do Nehab. Continuo sem ver como usar a expressão p(x) =
(x-1)(x-2)(x-3)Q(x) + ax^2 + bx + c ajuda a resolver a questão. A
divisão euclidiana que ele faz (conforme a outra mensagem dele na
lista) não garante que Q(x) tem coeficientes inteiros.

Abraços,
-- 
Bernardo Freitas Paulo da Costa

-- 
Esta mensagem foi verificada pelo sistema de antivírus e
 acredita-se estar livre de perigo.


=
Instruções para entrar na lista, sair da lista e usar a lista em
http://www.mat.puc-rio.br/~obmlistas/obm-l.html
=


[obm-l] Re: [obm-l] Re: [obm-l] Ternas pitagóricas

2014-02-22 Por tôpico Bernardo Freitas Paulo da Costa
2014-02-22 8:07 GMT-03:00  jjun...@fazenda.ms.gov.br:

 - Mensagem Original -
 [obm-l] Ternas pitagóricas

 Existe alguma terna pitagórica cujos dois menores termos
 são números consecutivos,além de (3,4,5)?

 Se não errei... há o terno: 20, 21 e 29.
 Outro é 119, 120 e 169.

Exato.

A equação é a^2 + (a+1)^2 = c^2. Usando a substituição z = a + 1/2, ela se torna

(z - 1/2)^2 + (z + 1/2)^2 = c^2
2z^2 + 1/2 = c^2
4z^2 + 1 = 2c^2

Chamando y = 2z, isso dá uma equação de Pell:

y^2 + 1 = 2c^2

Note que y é inteiro porque z é inteiro mais meio.

Essa equação tem infinitas soluções, por exemplo continuando as suas:

696^2 + 697^2 = 985^2
4059^2 + 4060^2 = 5741^2
23660^2 + 23661^2 = 33461^2
137903^2 + 137904^2 = 195025^2
803760^2 + 803761^2 = 1136689^2
4684659^2 + 4684660^2 = 6625109^2
27304196^2 + 27304197^2 = 38613965^2
159140519^2 + 159140520^2 = 225058681^2
927538920^2 + 927538921^2 = 1311738121^2
5406093003^2 + 5406093004^2 = 7645370045^2

Abraços,
-- 
Bernardo Freitas Paulo da Costa

-- 
Esta mensagem foi verificada pelo sistema de antivírus e
 acredita-se estar livre de perigo.


=
Instruções para entrar na lista, sair da lista e usar a lista em
http://www.mat.puc-rio.br/~obmlistas/obm-l.html
=


[obm-l] Re: [obm-l] Re: [obm-l] Re: [obm-l] Re: [obm-l] RE: [obm-l] Re: [obm-l] Três de inteiros

2014-02-21 Por tôpico Bernardo Freitas Paulo da Costa
2014-02-21 14:24 GMT-03:00 Tarsis Esau tarsise...@gmail.com:
 Fiz a segunda, vou tentar fazer a terceira :)

 m³ + n³ + 99mn = 33³

 (m + n)³ - 3m²n - 3mn² + 99mn = 33³
 (m + n)³ - 33³ = 3mn.[(m + n) - 33]
 [(m +n) - 33].[(m + n)² + (m +n).33 + 33²] = 3mn.[(m+n) - 33]

 Assim, temos

 1) m + n - 33 = 0

 e

Deveria ser ou, mas você agiu como se fosse ou. Mas isso é menos
importante que o meu próximo comentário.

 2) (m + n)² + (m + n).33 + 33² = 3mn

 De 1) temos todos os pares (x,y): (0,33); (1,32), ..., (32, 1), (33, 0).
 Todos os inteiros estão neste intervalo.

 Uma vez que , caso um m seja maior que 33, o n necessariamente deve ser
 menor que zero, o que vai contra o enunciado de m.n =0.

 Desse modo, não há necessidade de resolver 2).

Claro que há. Pode ser que a equação 2 tenha uma solução com m = 20 e
n = 10 (sei lá) cuja soma não é 33. Se você tivesse obtido TODOS os
pares (a,b) com 0=a=33, 0 =b=33 como solução, aí tava certo. Mas
veja que essa é uma equação cúbica, portanto para cada m existem
três soluções n possíveis. É bastante provável que, se m é inteiro,
não haja muitas soluções com n inteiro, mas você tem que demonstrar
isso. Além disso, o enunciado diz que m.n = 0, ou seja, pode ser que
m e n sejam NEGATIVOS! (mas talvez o enunciado tenha sido copiado
errado, e era para ser m E n = 0).
-- 
Bernardo Freitas Paulo da Costa

-- 
Esta mensagem foi verificada pelo sistema de antivírus e
 acredita-se estar livre de perigo.


=
Instruções para entrar na lista, sair da lista e usar a lista em
http://www.mat.puc-rio.br/~obmlistas/obm-l.html
=


[obm-l] Re: [obm-l] Re: [obm-l] RE: [obm-l] Re: [obm-l] Três de inteiros

2014-02-20 Por tôpico Bernardo Freitas Paulo da Costa
2014-02-20 19:47 GMT-03:00 saulo nilson saulo.nil...@gmail.com:
 2014-02-20 18:46 GMT-03:00 marcone augusto araújo borges
 marconeborge...@hotmail.com:

 2) Determine todos os pares de inteiros (m,n) tais que m.n  = 0 e
 m^3 + n^3 + 99mn = 33^3

 2--
 m+n=33
 3m^2n+3mn^2=99mn

 Na segunda (m+n)^3 = m^3 + n^3 + 3mn(m+n)
 Foi isso que vc viu?

 foi.

A idéia é muito boa, mas não acho que isso implique m+n = 33. Isso é
apenas uma solução (enfim, uma família). Mas pode ser que m+n != 33,
daí você pode cortar (33 - (m+n)) dos dois lados da equação e ficar
com uma equação quadrática

3mn = 33^2 + 33(m+n) + (m+n)^2. (subtraia (m+n)^3 dos dois lados; na
esquerda, expanda, cancele os cubos, fatore; na direita usa a^3 - b^3
= (a-b)(a^2 + ab + b^2), e veja que (a-b) =  (33 - (m+n)) aparece dos
dois lados).

E tem que obter todas as soluções nesse caso também. (Pode ser que não
haja nenhuma solução inteira, não pensei ainda!)

Abraços,
-- 
Bernardo Freitas Paulo da Costa

-- 
Esta mensagem foi verificada pelo sistema de antivírus e
 acredita-se estar livre de perigo.


=
Instruções para entrar na lista, sair da lista e usar a lista em
http://www.mat.puc-rio.br/~obmlistas/obm-l.html
=


Re: [obm-l] Primos

2014-02-18 Por tôpico Bernardo Freitas Paulo da Costa
2014-02-18 8:48 GMT-03:00 marcone augusto araújo borges
marconeborge...@hotmail.com:
 Determine os primos p tais que (p+1)/2 e (p^2 + 1)/2 são quadrados perfeitos

 p = 2k + 1 = (p+1)/2 = k+1
 k+1 = t^2 = k = t^2 - 1 = p = 2t^2 - 1
 (p^2 +1)/2 = 2t^4 - 2t^2 + 1 = m^2
 2t^4 - 2t^2 + 1 - m^2 = 0
 Delta =  4(2m^2 - 1) = 2m^2 - 1 = n^2
 Deu pra ver que m = 5(e n = 7) satisfaz
 Dai t = 2,k = 3 e p = 7(há outro valor para p?)
 Como resolver mesmo 2m^2 - 1 = n^2 ?

Isso dá uma equação de Pell. Mas acho que você talvez tenha que usar
que p é primo em algum lugar, talvez seja mais simples do que resolver
Pell.

Abraços,
-- 
Bernardo Freitas Paulo da Costa

-- 
Esta mensagem foi verificada pelo sistema de antivírus e
 acredita-se estar livre de perigo.


=
Instruções para entrar na lista, sair da lista e usar a lista em
http://www.mat.puc-rio.br/~obmlistas/obm-l.html
=


Re: [obm-l] Problema do Cavalo

2014-02-18 Por tôpico Bernardo Freitas Paulo da Costa
2014-02-18 14:30 GMT-03:00 Benedito bened...@ufrnet.br:

 É infinito nos quatro quadrantes, que é para permitir muitos movimentos.

 De: owner-ob...@mat.puc-rio.br [mailto:owner-ob...@mat.puc-rio.br] Em nome de 
 terence thirteen
 Enviada em: segunda-feira, 17 de fevereiro de 2014 08:16
 Para: obm-l
 Assunto: Re: [obm-l] Problema do Cavalo

 Ele é infinito nos quatro quadrantes?

 Eu tentaria algo como construir um grafo infinito, mas vou pensar antes...

Eu tenho uma idéia de solução no braço. Supondo que a questão seja:
Qual é o número de casas diferentes em que um cavalo pode terminar
uma seqüência de N movimentos. Assim, para n = 1, temos 8 casas
(brancas), e para n = 2 temos 33 casas (pretas, incluindo a casa preta
original!).

Para n maior, a seqüência fica assim (feito num computador, na marra):

8; 33; 76; 129; 196; 277; 372; 481; 604; 741; 892; 1057; 1236; 1429; ...

Agora, vem o chute principal (que é o que vai ajudar a gente a fazer
indução): Calcule as diferenças sucessivas dos elementos! Isso dá:

25; 43; 53; 67; 81; 95; 109; 123; 137; 151; 165; 179; 193; ...

Ainda não parece bom ? Não tem problema... Mais uma vez, faça as diferenças:

18; 10; 14; 14; 14; 14; 14; 14; 14; 14; 14; 14; ...

Ah ! Parece que é uma PA de segunda ordem, a partir de um
certo ponto...

Vamos entender essa idéia. No longo prazo, o cavalo vai se afastando
do centro, e portanto ele pode cobrir uma área no máximo proporcional
a N^2. Isso por si só já justifica tentar achar uma PA de segunda
ordem. O que é interessante é que a parte perto do centro (depois do
início, onde ainda há um monte de buracos meio aleatórios) estará
completamente coberta depois de um certo tempo, e o que interessa é o
que acontece nas coroas. Agora, tem que justificar que as coroas têm
uma espessura constante depois de passada a parte transiente
inicial.

Como eu usei um computador, e posso calcular mais do que n = 10 (por
exemplo n = 100) e os 14 continuam até esse ponto. Para mim, isso é
mais do que suficiente para eu ter certeza que a resposta é essa, mas
admito que falta um argumento garantindo que basta observar um número
finito de passos para acertar a recorrência. Eu diria que, como um
cavalo completa a vizinhança do ponto inicial (o 3x3 em volta da
origem) em uma quantidade finita de passos (basta chegar na
profundidade 3 do grafo do Torres) a recorrência não pode ser de ordem
muito maior do que isso. Para melhorar, veja que a partir de 3 passos,
o que temos é um octógono, TODO preenchido, dos quadrados brancos (que
são os únicos em que o cavalo pode estar!). Daí pra frente, não é
difícil ver que a cada etapa teremos um octógono com lado aumentando
de 1 a cada vez. Veja também que a partir do 3o termo da segunda
diferença, só tem 14. Não é coincidência.

Agora, eu deixo a indução para você completar!

Abraços,
-- 
Bernardo Freitas Paulo da Costa

-- 
Esta mensagem foi verificada pelo sistema de antivírus e
 acredita-se estar livre de perigo.


=
Instruções para entrar na lista, sair da lista e usar a lista em
http://www.mat.puc-rio.br/~obmlistas/obm-l.html
=


Re: [obm-l] duas identidades

2014-01-21 Por tôpico Bernardo Freitas Paulo da Costa
2014/1/21 Luís qed_te...@hotmail.com:
 Sauda,c~oes,

 Como mostrar que

 x^(2n) - 1 = (x^2-1)(x^(2n)+x^(2n-1)++1)=(x^2-1) X
 \prod_{k=1}^{n-1} (x^2 - 2x cos(k\pi/n) + 1)

 e

 x^(2n+1) = (x+1)(x^(2n)-x^(2n-1)++1)=(x+1) X
 \prod_{k=1}^{n} (x^2 - 2x cos((2k-1)/(2n+1)) + 1)

Olhe para as raízes complexas desses polinômios, e faça pares de
raízes conjugadas. Vou fazer o primeiro:

x^(2n) - 1 = 0 = x = exp(2 pi i * k /2n), para k = 0, 1, 2, … (2n -
1). Separe k = 0 e k = n, que dão as raízes x = 1 e x = -1, sobram as
raízes exp( +- 2 pi i * k / 2n) para k = 1, 2, … n-1 (usando que tudo
é periódico módulo 2n !!). Seja w = exp(2 pi i / 2n), agrupando os
fatores (x - w^k) e (x - w^(-k)) temos
(x^2 - (w^k + w^(-k))x + 1) = (x^2 - 2 cos(k * 2 pi/2n) x + 1).

Obs: a fatoração intermediária está errada, deveria começar em x^(2n -
2), para dar o grau certo.

Abraços,
-- 
Bernardo Freitas Paulo da Costa

-- 
Esta mensagem foi verificada pelo sistema de antivírus e
 acredita-se estar livre de perigo.


=
Instruções para entrar na lista, sair da lista e usar a lista em
http://www.mat.puc-rio.br/~obmlistas/obm-l.html
=


[obm-l] Re: [obm-l] Re: [obm-l] Re: [obm-l] Re: [obm-l] RE: [obm-l] Re: [obm-l] Limite de uma variável

2014-01-01 Por tôpico Bernardo Freitas Paulo da Costa
2014/1/1 Pacini Bores pacini.bo...@globo.com:
 Ok! Ralph, obrigado pela sua observação e explicação .

 Se tivesse dito : k 0   tão pequeno quanto eu queira tal que 0|x-a|k ,
 teria algum problema ?
Teria. Essa (e outras) frases de cálculo são recursos intuitivos úteis
para pensar sobre limites, mas não para definí-los. Assim, quando você
diz tão pequeno quanto eu queira, isso é uma abreviação para uma
frase bem mais complicada. E que, nesse caso (limites de uma única
variável) não faz sentido, porque o que esta abreviação contém é uma
relação dependência entre várias quantidades relacionadas ao
comportamento de DUAS variáveis juntas.

 Ou no momento que estou escrevendo  tão pequeno quanto eu queira, já estou
 definindo algo que k  depende ?
Na verdade, você está definindo alguma coisa que vai depender de k.
Nas definições habituais, o seu k é chamado de épsilon, e o delta
é que depende do épsilon quando eles aparecem.

Mas o maior problema, mesmo, como disse o Ralph, é que o limite de
alguma coisa só faz sentido de esta mesma coisa (o x) variar. Na sua
frase

para todo k, existe x tal que ...,

o x aparece depois do k, então ele não varia, ele existe.

Se você conhece programação, isso é exatamente o que acontece quando
você define uma variável local com o mesmo nome de uma variável
global. Daí pra frente, dentro do bloco onde você estiver, a
variável global está inacessível. Em matemática, você define uma
variável quando você a introduz numa fórmula por meio de um existe
ou um para todo. Assim, no seu exemplo, o lim x = a que vem antes
está falando de um x que NÃO é o mesmo que o que você introduz
depois por existe!

Uma outra forma de pensar é que os nomes das variáveis são totalmente
neutros. Ou seja, a sua frase não pode mudar de valor lógico se você
substituir todos os x de uma afirmação por y, ou z. Nesse seu caso,
o problema é que existem várias (sub-)afirmações dentro da definição
(que é o análogo exato dos blocos de código num programa) e portanto
em CADA uma delas, as variáveis novas poderiam ser chamadas como você
quiser.

E é exatamente por o seu x ser uma nova variável que o Ralph pode
dizer que a sua (sub)-afirmação era sempre verdadeira, qualquer que
fosse o a. A ordem em que você introduz as variáveis muda o sentido da
frase! (Ou seja, a fala não é comutativa ;-))

 Em 1 de janeiro de 2014 13:02, Ralph Teixeira ralp...@gmail.com escreveu:

 Desculpa, Pacini, mas isto nao faz sentido se voce nao disser algo sobre o
 que x significa. A frase que voce escreveu:

 para todo k0, existe x real tal que 0|x-a|k

 eh simplesmente VERDADEIRA, sempre -- SEMPRE existe esse x real, basta
 tomar x=a+k/2, por exemplo.

 2014/1/1 Pacini Bores pacini.bo...@globo.com

 Olá Pedro,

 Podemos definir o que desejas da seguinte forma : limx =a , com a real;

  para todo k0 , existe x  real tal que  0  |x - a|  k  .

Abraços, e bom 2014
-- 
Bernardo Freitas Paulo da Costa

-- 
Esta mensagem foi verificada pelo sistema de antivírus e
 acredita-se estar livre de perigo.


=
Instruções para entrar na lista, sair da lista e usar a lista em
http://www.mat.puc-rio.br/~obmlistas/obm-l.html
=


[obm-l] Re: [obm-l] RE: [obm-l] Re: [obm-l] Re: [obm-l] RE: [obm-l] Re: [obm-l] Limite de uma variável

2014-01-01 Por tôpico Bernardo Freitas Paulo da Costa
2014/1/1 Pedro Chaves brped...@hotmail.com:
 Muito obrigado, Ralph e Pacini.

 Continuo em dúvida:

 Como expressar em linguagem formal as afirmações x tende para a, x tende a 
 mais infinito e x tende a menos infinito?
 Como provar que as afirmações x tende a mais infinito e x + r tende a mais 
 infinito são equivalentes?  ( x é variável real e r é uma constante real) 
 —-- Questão já proposta na Lista.

Bom, as afirmações acima não são formais. O primeiro passo, portanto,
seria transformá-las em afirmações formais, dando um sentido preciso.
Assim, eu vejo duas formas.

Podemos pensar x como uma seqüência infinita de valores x_1, x_2,
... x_n, ... Daí, x+r será um abuso de notação para a seqüência
infinita (x_1 + r), (x_2 + r), ... , (x_n + r), ... . Então, a
demonstração será sobre limites de sequências.

A segunda forma é como fizeram antes: x é uma função de uma outra
variável (tempo, por exemplo, que é a metáfora mais comum para o
entendimento de limites), e neste caso x+r será uma outra função,
tomando valores r maiores. E a demonstração será, agora, sobre
limites de funções.


Talvez o que complique a coisa seja o seguinte. Existe uma expressão
informal para limites que é f(x) tende a A quando x tende a B, e
parece que precisamos dar um sentido separado para quando x tende a
B. É claro que fazendo isso, também temos um sentido separado para
f(x) tende a A, e assim acabamos de decompor uma sentença em duas.
Por mais que isso seja interessante e intuitivo (como frase do
português), o problema todo é que a expressão original não é
matematicamente formal. Ela abrevia uma coisa complicada (com épsilons
e deltas, desigualdades, para todos e existes) e, quando você vai
ver, não dá para separar. Como o Ralph e o Kelvin já escreveram, eu
vou pegar carona, repetir, e tentar mostrar que estas duas partes são,
realmente, inseparáveis. Pelo menos, como foram definidas. Talvez
valha a pena tentar dar uma definição formal do que seja x tende a B
sem ligar com mais nada, mas até hoje ninguém achou uma.

Seja f uma função definida num intervalo aberto contendo B. (B poderia
estar na borda, poderia ser mais geral do que um intervalo, mas isso
não vem ao caso aqui)
A expressão A é o limite de f(x) quando x tende a B quer dizer, exatamente:

Para todo épsilon positivo, existe delta positivo, tal que, para todo
x no intervalo, se |x - B|  delta então | f(x) - A |  épsilon

Note que a expressão original é uma afirmação com 3 variáveis livres:
f, A e B.  O x é uma variável muda da definição: isso é mais claro
ao ler a versão formal, onde eu introduzi o x com o para todo na
frente, e poderia ter chamado de w que não ia fazer a menor
diferença (entre a afirmação ser verdadeira ou falsa claro ;  você
pode achar - e eu concordo - que chamar uma coisa de x dá uma idéia
diferente de chamar a mesma coisa de batata ou w).

A parte que parece que tem a ver com o x tende a B é a seguinte:

existe delta positivo, tal que, para todo x no intervalo, se |x - B|  delta

Fora disso, parece que tem mais a ver com o f(x) tende a A, não?
Mas o problema é que esta frase que a gente obteve ficou capenga, ela
não pede muita coisa para o x, nem explica muita coisa também. Ela
diz: x está no intervalo e bota um se, mas não completa o então.
Pior ainda, o então | f(x) - A |  épsilon, na verdade, é uma
condição no x. Isso é até mais claro quando você bota os parêntesis na
frase para interpretar direito:

Para todo épsilon positivo ( existe delta positivo, tal que [ para
todo x no intervalo, { se |x - B|  delta, então | f(x) - A | 
épsilon } ] )

Assim, você não pode separar a parte de dentro como sendo uma das
metades da definição. Você até poderia cortar a frase para ficar só
com os parêntesis (e você teria uma afirmação com 4 variáveis livres,
f, A, B e épsilon), ou para ficar só com os colchetes (e daí seriam 5
variáveis, porque aparece o delta) ou só com as chaves (e daí você tem
6 variáveis, f, A, B, épsilon, delta, x, na ordem em que foram
introduzidas, o A e o B introduzidas simultaneamente). E veja que, até
o nível mais profundo da definição (o com 6 variáveis) o f aparece. O
que explica porque não dá para separar direito.


Espero que ajude... um dos grandes problemas da análise (e que levou
sua cota de séculos para ser resolvido) foi justamente esse de passar
de uma idéia intuitiva de limites de uma variável para a definição
formal de limites de uma coisa DEPENDENDO do comportamento de outra.
A vantagem da primeira é que você dá sentido às duas partes da frase
limite de f(x) quando x tende a B, mas o problema é que, como é
apenas intuitivo, você não consegue fazer uma demonstração 100%
formal.

Abraços,
-- 
Bernardo Freitas Paulo da Costa

-- 
Esta mensagem foi verificada pelo sistema de antivírus e
 acredita-se estar livre de perigo.


=
Instruções para entrar na lista, sair da lista e usar a lista em
http://www.mat.puc-rio.br/~obmlistas/obm-l.html
=


Re: [obm-l] problema

2013-12-14 Por tôpico Bernardo Freitas Paulo da Costa
2013/12/13 Artur Steiner artur_stei...@hotmail.com:
 Como a exponencial é sempre positiva, não há solução negativa. Para x = 0, 
 definamos f(x) = 2^x - x, de modo que f(0) = 1 e f'(x) = 2^x ln (2) - 1. Como 
 ln(2)  0, f' é estritamente crescente, logo f é convexa. f' se anula em x* 
 tal que 2^x* = 1/ln(2). Como ln(2) está em (0, 1), 1/ln(2)  1 e x*  0. 
 Logo, f tem um mínimo global em x*, que está no eixo real positivo.

 Temos que min f = f(x*) = 1/ln (2) - (ln(1/ln(2)))/ln(2) = 1/ln(2) + 
 ln(ln2))/ln(2) = (1 + ln(ln2)))/ln(2)

 Com uma planilha Excel, verifiquei que f(x*)  0. Logo, esta equação não tem 
 solução real.
f(x*) = [1 + ln(ln(2)) ] / ln(2) é maior do que 0 = 1 + ln(ln(2)) 
0 = ln(ln(2))  -1 = ln(2)  exp(-1) = 1/2.7818...

Exponencial de novo: sse 2  exp(1/2.7181...), e basta ver que 2 
exp(1/2) = 2  1 + 1/2 + 1/2*1/4 + 1/3!*1/8 + ..., o que é verdade
porque, sem os fatoriais no denominador, seria igualdade! (e todos os
termos são positivos)

Abraços,
-- 
Bernardo Freitas Paulo da Costa

-- 
Esta mensagem foi verificada pelo sistema de antivírus e
 acredita-se estar livre de perigo.


=
Instruções para entrar na lista, sair da lista e usar a lista em
http://www.mat.puc-rio.br/~obmlistas/obm-l.html
=


[obm-l] Re: [obm-l] Re: [obm-l] Re: [obm-l] Re: [obm-l] Aritmética

2013-12-06 Por tôpico Bernardo Freitas Paulo da Costa
2013/12/4 Cassio Anderson Feitosa cassiofeito...@gmail.com:
 Mas acredito que o outro raciocínio levou a todas as soluções.
É. Quando p é um número primo, uma equação do segundo grau n^2 = x
(mod p) ou tem duas raízes, ou não tem nenhuma (o único caso de raiz
única é n^2 = 0, mas isso é uma raiz dupla). No seu caso, para
mostrar que n^2 = x (mod p*q), você tem que mostrar que cada uma das
equações tem raiz, logo terá duas, e portanto ao fazer as combinações,
você obtém 4 raízes ao todo (mod p*q). Basta que haja uma para que
haja infinitas, mas se houver uma raíz mod (p*q) então você terá 4
(com multiplicidades, se for o caso). É um exercício legal calcular
quantas dessas equações n^2 == x (mod p*q) tem 4 soluções diferentes,
2 soluções duplas, 1 solução quádrupla (x = 0 apenas) e deduzir
quantas delas não têm !

 Em 4 de dezembro de 2013 13:50, Cassio Anderson Feitosa
 cassiofeito...@gmail.com escreveu:

 8n^2+5\equiv 0\pmod 77  é equivalente a 8n^2+5 == 0 mod 7e
 8n^2+5== 0 mod 11.

 Primeira parte: 8n² == 5 mod 11  == 8n^2 == 6mod 11
Aqui teve um errinho (e também mais embaixo) que me comeu um tempo...
de 8n^2 + 5 == 0, você esquece o sinal e faz 8n^2 == 5, (em vez de ==
-5) e depois você inverte o sinal 8n^2 == 6 e acerta de novo.

Fora isso, certíssimo. Você poderia ter invertido 8 módulo 11 (8*7 =
56 == 1) e obter um pouco mais rápido 0 = 7*8n^2 + 7*5 == 56 n^2 + 35
== n^2 + 2 = n^2 == -2 == 9 mod 11. (O mesmo vale embaixo, onde 8 ==
1 mod 7 direto, 0 == 8n^2 + 5 == n^2 + 5 = n^2 == -5 == 2 == 9 mod 7)

 == 4n² == 3 mod
 11 == 3(4n²) == 9 mod 11 ==  12n²==n²==9 mod 11 ===n==3 ou n== -3
 mod 11, ou seja, n==3 ou n== 8 mod 11.

  Segunda parte: 8n² == 5 mod 7 == 8n^2 == 2mod 7 == 4n² == 1 mod 7
 == 2(4n²) == 2 mod 7 ==  8n²==n²==2 mod 7. =   n==3 ou n== -3 mod
 11, ou seja, n==3 ou n== 4 mod 7.

  Então, o sistema n == 3 mod 11 e n == 3 mod 7 gera uma solução.
  o sistema  n == 3 mod 11 e n == 4 mod 7 gera outra solução
 n == 8 mod 11 e n == 3 mod 7 outra solução
 n == 8 mod 11 e n == 4 mod 7 outra solução.


  Daí basta pegar cada sistema de duas congruências e resolver pelo
 Teorema chinês de Resto.

  Por exemplo, a solução pro primeiro sistema é n=77q + 3, q inteiro.

 --
 Cássio Anderson
 Graduando em Matemática - UFPB

-- 
Bernardo Freitas Paulo da Costa

-- 
Esta mensagem foi verificada pelo sistema de antivírus e
 acredita-se estar livre de perigo.


=
Instruções para entrar na lista, sair da lista e usar a lista em
http://www.mat.puc-rio.br/~obmlistas/obm-l.html
=


[obm-l] Re: [obm-l] Transformação Linear

2013-11-28 Por tôpico Bernardo Freitas Paulo da Costa
2013/11/28 João Sousa starterm...@hotmail.com:
 Considere a transformação linear A: R3 - R4, de forma que v = (2, -1,1)
 esteja no núcleo e
 que B = {(1, 2, -1, 0), (3, 0, 1, 2)} seja uma base de sua imagem. Então, A
 (3, 2,1) é igual a

Bom, sejam v1 e v2 tais que A(v1) = (1, 2, -1, 0) e A(v2) = (3, 0, 1,
2). Escrevendo (3,2,1) = a*v + b*v1 + c*v2, temos que

A(3,2,1) = b* (1, 2, -1, 0) + c * (3, 0, 1, 2).

O problema é que achar v1 e v2 é impossível com as informações do
enunciado. Talvez v1 = (1,0,0) e v2=(0,1,0), mas sei lá.

 (A) (10, 2, 2, 6)
 (B) (10, 2, 6, 2)
 (C) (2, 10, 2, 6)
 (D) (2, 2, 6, 10)
 (E) (6, 2, 10, 2)

Mas tudo isso não importa. Enfim, importar... sei lá. O fato é que
apenas o vetor da resposta (A) está no subespaço gerado pela base B,
então essa é a única resposta possível dentre estas. Por sinal, isso
quer dizer que v1 = (1,0,0) e v2=(0,1,0), mas note que QUALQUER
escolha de dois vetores v1 e v2 tais que {v, v1, v2} seja uma base do
R3 dá uma transformação linear A satisfazendo as condições do
enunciado. Ou seja, QUALQUER vetor no subespaço gerado por B seria uma
resposta válida.

Abraços,
-- 
Bernardo Freitas Paulo da Costa

-- 
Esta mensagem foi verificada pelo sistema de antivírus e
 acredita-se estar livre de perigo.


=
Instruções para entrar na lista, sair da lista e usar a lista em
http://www.mat.puc-rio.br/~obmlistas/obm-l.html
=


Re: [obm-l] Coordenadas polares

2013-11-24 Por tôpico Bernardo Freitas Paulo da Costa
2013/11/24 marcone augusto araújo borges marconeborge...@hotmail.com

 Alguém poderia mostrar como fica o gráfico de rô = sec(teta)?

Transforme r e teta em coordenadas cartesianas, r^2 = x^2 + y^2,
tan(teta) = y/x, e depois substitua um pouco de trigonometria.

Abraços,
-- 
Bernardo Freitas Paulo da Costa

-- 
Esta mensagem foi verificada pelo sistema de antivírus e
 acredita-se estar livre de perigo.


=
Instruções para entrar na lista, sair da lista e usar a lista em
http://www.mat.puc-rio.br/~obmlistas/obm-l.html
=


Re: [obm-l] Coordenadas polares

2013-11-24 Por tôpico Bernardo Freitas Paulo da Costa
2013/11/24 marcone augusto araújo borges marconeborge...@hotmail.com:
 r = sec(teta)
 r^2 = [sec(teta)]^2 = 1 + [tg(teta)]^2
 x^2 + y^2 = 1 + (y/x)^2
 E dai?

Daí que

x^2 + y^2 = 1 + (y/x)^2 = (x^2 + y^2)/x^2, logo x^2 = 1 (pois r != 0,
já que a secante é sempre diferente de zero, logo podemos cortar x^2 +
y^2). Daí, x = 1 ou x = -1, e mais um pouco de trigonometria vai
mostrar que x = -1 é uma solução espúria, introduzida porque a gente
elevou ao quadrado (isso quase sempre acontece, e é sempre bom
verificar como... aqui, se fosse r = - sec(teta), teria dado a mesma
depois de elevar ao quadrado, portanto claro que uma das soluções tem
que ser desprezada). Isso quer dizer que x = 1, ou seja, e é equação
de uma reta vertical.

Abraços,
-- 
Bernardo Freitas Paulo da Costa

-- 
Esta mensagem foi verificada pelo sistema de antivírus e
 acredita-se estar livre de perigo.


=
Instruções para entrar na lista, sair da lista e usar a lista em
http://www.mat.puc-rio.br/~obmlistas/obm-l.html
=


Re: [obm-l] contagem

2013-11-20 Por tôpico Bernardo Freitas Paulo da Costa
2013/11/20 PeterDirichlet peterdirich...@gmail.com

 On 20-10-2013 16:28, marcone augusto araújo borges wrote:

 Quantas matrizes 4 x 4 formadas pelos elementos 1,2,3 e 4 possuem
 em cada linha e em cada coluna todos elementos distintos?


 Vou testar uma ideia.

 Sabemos que, se trocarmos 1,2,3,4 por qualquer permutação deles, de forma 
 consistente, obtemos outra matriz que satisfaz o enunciado.

 Por exemplo:

 1 2 3 4
 2 1 4 3
 3 4 1 2
 4 3 2 1

 Substitua 1 por 3 e 3 por 1, mantendo o 2 e o 4:

 3 2 1 4
 2 3 4 1
 1 4 3 2
 4 1 2 3

 Quando é que duas dessas matrizes seriam iguais? Nunca, pois a primeira linha 
 já seria diferente. Assim, de uma matriz obteremos outras 4! matrizes.

 Assim sendo, eu vou tacitamente aceitar que a primeira linha é '1 2 3 4', e 
 depois multiplicar por 4!.

 Mais uma coisa interessante é que podemos permutar as linhas entre si! Veja a 
 segunda matriz:

 1 2 3 4
 3 4 1 2
 4 3 2 1
 2 1 4 3

 É óbvio conferir que no caso geral a propriedade se manterá.

 Assim, eu posso pensar que a 'borla' da matriz é assim:

 1 2 3 4
 2 x x x
 3 x x x
 4 x x x

 Basta depois multiplicar por 4! * 3!.

 Daqui para diante, me parece que complica um pouquinho...

 MAS eu não desconfiaria se a resposta não tiver a ver com permutações 
 cíclicas, no seguinte sentido:
 cada linha é permutação cícilca da primeira.

As quatro matrizes satisfazendo todas as condições (únicos elementos,
fixar os bordos) são:

1 2 3 4
2 1 4 3
3 4 1 2
4 3 2 1

1 2 3 4
2 1 4 3
3 4 2 1
4 3 1 2

1 2 3 4
2 3 4 1
3 4 1 2
4 1 2 3

1 2 3 4
2 4 1 3
3 1 4 2
4 3 2 1

Eu vou tentar escovar o programa para o caso 5x5, por enquanto ele
calcularia 5 ^ 16 casos...

-- 
Bernardo Freitas Paulo da Costa

-- 
Esta mensagem foi verificada pelo sistema de antivírus e
 acredita-se estar livre de perigo.


=
Instruções para entrar na lista, sair da lista e usar a lista em
http://www.mat.puc-rio.br/~obmlistas/obm-l.html
=


Re: [obm-l] Conjecturas especiosas

2013-10-30 Por tôpico Bernardo Freitas Paulo da Costa
2013/10/30 Hermann ilhadepaqu...@bol.com.br

 É possível montar essa série no EXCEL? Claro para os primeiros dígitos.
Montar, sim. Com precisão para PROVAR que os primeiros dígitos são o
que o Excel acha (ou qualquer outro software simples de contas),
provavelmente, não. Porque o Excel não prova nada ;-)

Vamos ver então como provar que algumas contas não vão dar muito errado !

Note que os termos são exp(-n^2 / 10^10). Ou seja, quando n é
pequeno (leia-se, tal que n^2  10^10) temos que os termos são,
praticamente, iguais a 1. Para que os termos se tornem pequenos, é
preciso que n^2 seja razoavelmente maior do que 10^10. Note que
exp(-1)  1/3, logo você com certeza vai ter que somar, pelo menos, um
monte de termos até 10^5 (e até -10^5 do outro lado). Só isso pode já
ser um problema de caber no Excel.

Eu acho (só acho) que a partir de n = 10^6 o resto será
suficientemente pequeno para ser desprezado. Isso tem a ver com
(a+b)^2 = a^2 + 2*a*b + b^2 : se b for muuito menor do que a, os
primeiros dígitos serão os de a^2.

Aqui, eu tomaria a = soma dos termos com |n|  10^6, b = cauda da série.

Restam, é claro, os problemas de precisão: somar 2 * 10^6 números de
ponto flutuante, sem contar que cada conta exp(-n^2 / 10^10) já tem um
erro embutido (exceto para n = 0, onde dá 1 exatamente, e o computador
acerta) pode gerar um erro relativo grande, algo como eps*2*10^6 (eps
= precisão da máquina = 2^-53). Isso quer dizer que o a tem um erro
deste tamanho, e portanto a^2 terá um erro também, de novo pela
fórmula (a+b)^2 mais ou menos igual ao dobro do erro. Assim, deve
dar algo como 8-10 dígitos exatos ao fazer essa conta.

A única coisa que falta é provar que o rabicho da série a partir de
10^6 + 1 é realmente pequeno. Comparando com a integral, temos
rabicho = b  1/10^5 * integral de 10^6 até infinito de exp(-n^2 /
10^10), substitua n = 10^5 * x,
 = 10^5 / 10^5 * integral de 10 até infinito de
exp(-x^2) = erfc(10) ~ 2*10^(-45)

(nota: essa integral não tem primitiva bonitinha, e daí nasceram as
erf e erfc, que foram bastante estudadas para justamente ter
estimativas legais quando aparecem, e serem calculadas de forma
eficiente

Se você não conhece as erfc - ou não tem acesso a um cara que calcule
ele -, você pode estimar a integral assim:

int_M^infinito exp(-n^2) dn  int_M^infinito(-M*n) dn = 1/M *
exp(-M*M), o que no nosso caso dá que erfc(10)  exp(-100)/10 que é
realmente bem menor do que 1 para ser desprezado; na verdade, é menor
do que o eps da máquina, que é 2^(-53), e portanto a partir daí não
faz mais diferença nenhuma.)

Abraços,
-- 
Bernardo Freitas Paulo da Costa

-- 
Esta mensagem foi verificada pelo sistema de antivírus e
 acredita-se estar livre de perigo.


=
Instruções para entrar na lista, sair da lista e usar a lista em
http://www.mat.puc-rio.br/~obmlistas/obm-l.html
=


Re: [obm-l] Conjecturas especiosas

2013-10-30 Por tôpico Bernardo Freitas Paulo da Costa
Sem muita análise de precisão das contas, um programa em C acha os
seguintes valores

somando entre +- k*10^5 :

k = 1  série =   2.23099613103075889, erro =0.91059652255903445
k = 2  série =   3.11227096594849684, erro =0.02932168764129647
k = 3  série =   3.14145386081124123, erro =0.00013879277855176
k = 4  série =   3.14159255672422244, erro =0.0009686557082
k = 5  série =   3.14159265358006223, erro =0.000973097
k = 6  série =   3.14159265358972117, erro =0.07216
k = 7  série =   3.14159265358972117, erro =0.07216

note que para k a partir de 6, os erros de truncamento (que eu não me
dei ao trabalho de eliminar) dominam e não adianta mais somar nada.
Tem também a ver que a cauda da série fica menor do que o epsilon da
máquina, mas nesse caso deveria haver um erro de no máximo 1 (ou 2, se
dermos azar) casa decimal, não 4 como vemos aqui.

Abraços,
-- 
Bernardo Freitas Paulo da Costa

-- 
Esta mensagem foi verificada pelo sistema de antivírus e
 acredita-se estar livre de perigo.


=
Instruções para entrar na lista, sair da lista e usar a lista em
http://www.mat.puc-rio.br/~obmlistas/obm-l.html
=


Re: [obm-l] Conjecturas especiosas

2013-10-30 Por tôpico Bernardo Freitas Paulo da Costa
2013/10/30 Hugo Tadashi tada...@gmail.com

 Esta eu vi em [1]: Com um computador pode-se verificar que os primeiros 42
 bilhões dígitos do pi e da série abaixo são iguais,

 [image: Inline image 1]
 mas a identidade é falsa.


 [1] Borwein, Jonathan M., and Peter B. Borwein. Strange series and high
 precision fraud. *The American mathematical monthly* 99.7 (1992):
 622-640.

 Abraços,
 Tadashi

Humpf, eu devia ter adivinhado... quanto maior for N (= 10^5 nesse caso),
mais próxima estará a fórmula de Pi. Ou, melhor dizendo:

lim N-infinito (1/N * soma exp(-n^2 / N^2) )^2 = Pi

Seria interessante provar (analiticamente) uma estimativa da diferença em
função de N.

Viva Riemann,
-- 
Bernardo Freitas Paulo da Costa

-- 
Esta mensagem foi verificada pelo sistema de antivírus e
 acredita-se estar livre de perigo.

pi.png

Re: [obm-l] Algarismos desconhecidos

2013-10-28 Por tôpico Bernardo Freitas Paulo da Costa
2013/10/28 marcone augusto araújo borges marconeborge...@hotmail.com

 O número 88x2 é o produto de 3 números pares consecutivos.
 Cada uma das letras x representa um algarismo.Determine cada um
 dos algarismos desconhecidos.
Força bruta: tire a raiz cúbica (porquê?) de 8852, vai dar
445.73 Portanto, tente 444*446*448, que dá certo: 88714752

Abraços,
-- 
Bernardo Freitas Paulo da Costa

-- 
Esta mensagem foi verificada pelo sistema de antivírus e
 acredita-se estar livre de perigo.


=
Instruções para entrar na lista, sair da lista e usar a lista em
http://www.mat.puc-rio.br/~obmlistas/obm-l.html
=


Re: [obm-l] funcao implicita e geogebra

2013-10-04 Por tôpico Bernardo Freitas Paulo da Costa
2013/10/4 Luís Lopes qed_te...@hotmail.com:
 Sauda,c~oes,
Oi Luís,

 Continuo sem saber como calcular a equação que fornece
 os pontos extremos (max e min) da curva mas talvez a
 teoria se encontre nos livros que tratam das Curvas Algébricas Planas.
Exato, mas não necessariamente desta forma.

Você tem uma equação implícita P(x,y) = 0, onde P é um polinômio (de
grau 3) nas duas variáveis. Se eu entendi o problema, você quer achar
o(s) ponto(s) desta curva com a maior ordenada possível.

Para começar, pense que a curva definida pela equação P(x,y) = 0 é
bonitinha (sem pontos duplos, sem singularidades, etc). Assim, imagine
uma parametrização local da curva: (x(t), y(t)). Assim, pontos de
máximo (e mínimo, ou de inflexão) são dados por dy/dt = 0.

Agora, note que P(x(t),y(t)) = 0 em todos os pontos da curva, logo
para todos os t da sua parametrização. Daí:
dP/dx(x(t), y(t))*dx/dt + dP/dy(x(t), y(t))*dy/dt = 0.

Se dy/dt = 0, temos então que dP/dx(x(t), y(t)) * dx/dt = 0. Como a
curva é lisa (e sem singularidades, pontos duplos, etc, etc), dx/dt
não pode ser = 0. Daí, os pontos de máximo também satisfazem uma
equação suplementar, dP/dx(x,y) = 0.

Agora, você tem que achar as soluções do sistema {P(x,y) = 0,
dP/dx(x,y) = 0}. No seu caso particular, dP/dx é de grau 2, logo você
pode escrever y em função de x, e substituir na equação de grau 3...

Mas, em geral, existe uma teoria para achar as soluções de sistemas de
equações simultâneas, que são os resolventes, cf
http://en.wikipedia.org/wiki/Resultant e
http://en.wikipedia.org/wiki/Sylvester_matrix, em particular a parte
de aplicações a interseções do resultante. Essencialmente, eles
substituem uma equação na outra, de forma algorítmica, e dão
diretamente a equação que todas (e não metade, como seria o caso se
você pegasse cada uma das soluções da eq do segundo grau) as abscissas
possíveis y satisfazem. Os tais discriminantes são o caso particular
do resolvente de um polinômio e de sua derivada. Note que isso envolve
considerar P(x,y) como um polinômio em x cujos coeficientes são
polinômios em y. Os livros de Curvas Algébricas em geral, vão falar
deste parágrafo apenas (provar que a Resultante realmente faz a
eliminação das variáveis, como calcular, como que os discriminantes
têm a ver com resultantes).

Exemplinho: Seja P(x,y) = (x^2 + x y^2 + x y + 2y + 2) = x^2 + (y^2 +
y)*x + (2y + 1). Chame a = 1, b = (y^2 + y), c = (2y + 1). Nesse caso,
o discriminante é o usual, ou seja, b^2 - 4*a*c = (y^4 + 2y^3 + y^2)
- 8y - 4. As raízes disso dão os y máximos e mínimos locais.
Dá pra ver tudo isso com o WolframAlpha

http://www.wolframalpha.com/input/?i=+x^2+%2B+%28y^2+%2B+y%29*x+%2B+%282y+%2B+1%29+%3D+0
http://www.wolframalpha.com/input/?i=%28y^4+%2B+2y^3+%2B+y^2%29+-+8y+-+4+%3D+0
http://www.wolframalpha.com/input/?i=discriminant%28+x^2+%2B+%28y^2+%2B+y%29*x+%2B+%282y+%2B+1%29%2C+x%29


Voltando ao seu problema, o WA dá o discriminante:
http://www.wolframalpha.com/input/?i=discriminant%28%28-s^2y%2Bx^2y%2By^3%29Cos[A]%2B%28-s^2x%2B2sx^2-x^3%2B2sy^2-xy^2%29Sin[A]%2C+x%29

que é de grau 6 (como esperado de uma interseção de uma curva de grau
3 com uma curva de grau 2), mas que tem um fator y^2, que deve
provavelmente ser excluído do problema (certamente, não é o ponto de
máximo!). O fator que sobra deve ser o polinômio de grau 4 que
passaram pra você.

O mais chato é que o desenho da curva CC é meio feio
http://www.wolframalpha.com/input/?i=%28-10%C2%B2+y+%2B+x%C2%B2+y+%2B+y%C2%B3%29+11+%2F+14+%2B+%28-10%C2%B2+x+%2B+2+%2810%29+x%C2%B2+-+x%C2%B3+%2B+2+%2810%29+y%C2%B2+-+x+y%C2%B2%29+5+sqrt%283%29+%2F+14+%3D+0+
porque ela tem uma componente que vai pro infinito... Mas talvez seja
ela que você quer ? Para ter uma única solução real?

Além disso, o caso numérico, mais uma vez, dá
http://www.wolframalpha.com/input/?i=discriminant+%28+%28-10%C2%B2+y+%2B+x%C2%B2+y+%2B+y%C2%B3%29+11+%2F+14+%2B+%28-10%C2%B2+x+%2B+2+%2810%29+x%C2%B2+-+x%C2%B3+%2B+2+%2810%29+y%C2%B2+-+x+y%C2%B2%29+5+sqrt%283%29+%2F+14%2C+x%29
para o discriminante. Como a gente já sabe que tem um fator y^2, tem
no mínimo 4 soluções. De novo, o WA confirma a resposta:
http://www.wolframalpha.com/input/?i=-%284+%2849+sqrt%283%29+y^6%2B3300+y^5%2B17600+sqrt%283%29+y^4-247500+y^3-187500+sqrt%283%29+y^2%29%29%2F%2849+sqrt%283%29%29+%3D+0dataset=

Abraços,
-- 
Bernardo Freitas Paulo da Costa

(tenho que aprender a botar contas do WA sem que ele faça as contas a
cada vez, mas o problema é que o Clip fica apenas o resultado...)

-- 
Esta mensagem foi verificada pelo sistema de antivírus e
 acredita-se estar livre de perigo.


=
Instruções para entrar na lista, sair da lista e usar a lista em
http://www.mat.puc-rio.br/~obmlistas/obm-l.html
=


Re: [obm-l] Irracional entre dois racionais

2013-09-16 Por tôpico Bernardo Freitas Paulo da Costa
2013/9/16 Hermann ilhadepaqu...@bol.com.br

 Fazendo por diferenciais dá 0,125 e aí acredito que seja 0,13 a resposta.
Tem alguma razão para você ter sub-estimado a diferença? Veja que a
raiz quadrada é côncava, então a diferencial primeira superestima as
diferenças... Mas você acaba fazendo uma diferença de diferenças,
então sei lá. (Eu expandi em 8 = raiz(64))


 - Original Message -

 O número raiz(65) - raiz(63) está mais próximo de:

 a) 0,12   b) 0,13   c) 0,14   d) 0,15   e) 0,16

-- 
Bernardo Freitas Paulo da Costa

-- 
Esta mensagem foi verificada pelo sistema de antivírus e
 acredita-se estar livre de perigo.


=
Instruções para entrar na lista, sair da lista e usar a lista em
http://www.mat.puc-rio.br/~obmlistas/obm-l.html
=


Re: [obm-l] Irracional entre dois racionais

2013-09-16 Por tôpico Bernardo Freitas Paulo da Costa
2013/9/16 marcone augusto araújo borges marconeborge...@hotmail.com
 Tudo bem.Se não for abusar da boa vontade de vocês,eu gostaria de uma solução 
 que utilizasse assuntos do ensino médio.

raiz(a) - raiz(b) = (a - b)/(raiz(a) + raiz(b)). No seu caso, isso dá
2/(raiz(65) + raiz(63)). Daí, você tem que usar alguma forma de
chute... Não sei se vale usar concavidade, mas se for o caso, note
que raiz quadrada é côncava, então, raiz(65) + raiz(63)  2*raiz(64) =
2*8. Então, 2/(...)  2/(2*8) = 1/8 = 0.125.

Só não tenho muita certeza de como fazer sem derivadas (que
deveriam, sim, ser assunto de ensino médio... mas isso é outro debate)
para mostrar que é menor do que 0.135. Qualquer forma de estimativa
de erro vai acabar usando tangentes, derivadas segundas, etc.

Se você tiver paciência e um pouco de braço, você mostra que

2 / (raiz(65) + raiz(63))  0.135 elevando ao quadrado duas vezes para
eliminar os radicais, usando que 0.135 = 1/8 + 1/100 para fazer contas
com números razoáveis, depois que 63*65 + 1 = 64*64. Dá um certo
trabalho... tipo 1/3 de página.

Mas é justamente por isso que o Cálculo existe: para não ter que fazer
montes de contas braçais sem saber muito bem para onde vamos.

-- 
Bernardo Freitas Paulo da Costa

-- 
Esta mensagem foi verificada pelo sistema de antivírus e
 acredita-se estar livre de perigo.


=
Instruções para entrar na lista, sair da lista e usar a lista em
http://www.mat.puc-rio.br/~obmlistas/obm-l.html
=


[obm-l] Re: [obm-l] Re: Função periódica.

2013-09-16 Por tôpico Bernardo Freitas Paulo da Costa
2013/9/16 Francisco Lage franciscou...@gmail.com:
 Alguem?

 Em 15/09/2013 14:17, Francisco Lage franciscou...@gmail.com escreveu:

 Alguém pode me ajudar?


 Em 14 de setembro de 2013 15:51, Francisco Lage franciscou...@gmail.com
 escreveu:


 Alguem pode me ajudar ?
 --


 Francisco Lage




 --


  Francisco Lage
 ITA  T -16


 --
 Esta mensagem foi verificada pelo sistema de antivírus e
 acredita-se estar livre de perigo.

As mensagens estão vindo vazias. Não use imagens, não use anexos. Use
texto. Puro, com o mínimo de símbolos matemáticos estranhos (tipo um
quadrado que é um dois pequenininho em cima do x, nem todo mundo
pode ver isso: x²).

-- 
Bernardo Freitas Paulo da Costa

-- 
Esta mensagem foi verificada pelo sistema de antivírus e
 acredita-se estar livre de perigo.


=
Instruções para entrar na lista, sair da lista e usar a lista em
http://www.mat.puc-rio.br/~obmlistas/obm-l.html
=


[obm-l] Re: [obm-l] Função periodica

2013-09-16 Por tôpico Bernardo Freitas Paulo da Costa
2013/9/16 Francisco Lage franciscou...@gmail.com:
 Alguém pode me ajudar?

 Seja F : R - R*+ , uma função continua e periódica de período T , prove que
 (1/T)*inegral(f(x)/f(x+b))dx de 0 até 1 é maior ou igual a T , para todo b
 real

Isso tá meio errado... se f(x) = 1 para todo x, então a integral dá
1/T... Não seria 1/T * (integral de 0 até T) = 1 ?
-- 
Bernardo Freitas Paulo da Costa

-- 
Esta mensagem foi verificada pelo sistema de antivírus e
 acredita-se estar livre de perigo.


=
Instruções para entrar na lista, sair da lista e usar a lista em
http://www.mat.puc-rio.br/~obmlistas/obm-l.html
=


Re: [obm-l] Irracional entre dois racionais

2013-09-16 Por tôpico Bernardo Freitas Paulo da Costa
2013/9/16 Esdras Muniz esdrasmunizm...@gmail.com

 ( raiz(65)-raiz(63) )( raiz(65)+raiz(63) ) = 2
 mas raiz( (65+63)/2)(raiz(65) + raiz(63))/2 por MA=MQ
 ==
 raiz(65) - raiz(63)1/8=0,125
 raiz(65) - raiz(63),14=
 20,14(raiz(65) + raiz(63))0,14*(8+7)=2,1

Tinha que ser 0,135 e não 0,14. Ainda assim, suas estimativas dão
certo, porque 0.135 * (8+7) (ótima idéia) dá mais do que 2. Usando a
minha representação 1/8 + 1/100, dá:

(1/8 + 1/100)*(8 + 7) = 1 + 7/8  + 15/100 = 1 + 7/8 + 3/20 = 1 + (35 +
6)/40 = 2 + 1/40

Abraços,
-- 
Bernardo Freitas Paulo da Costa

-- 
Esta mensagem foi verificada pelo sistema de antivírus e
 acredita-se estar livre de perigo.


=
Instruções para entrar na lista, sair da lista e usar a lista em
http://www.mat.puc-rio.br/~obmlistas/obm-l.html
=


Re: [obm-l] Primos

2013-09-11 Por tôpico Bernardo Freitas Paulo da Costa
2013/9/11 marcone augusto araújo borges marconeborge...@hotmail.com

 Seja S um conjunto de primos tal que a,b E S(a e b não precisam ser 
 distintos) implica (ab+4) E S
 Mostre que S tem que ser vazio.

 Parece que há algo errado com o enunciado
 3 e 5 são primos e 3.5+4 =  19 é primo.
 Uma opinião?
Bom, note que como 5 e 19 estarão em S, daí 5*19 + 4 = 99 também. Mas
99 não é primo.

Abraços,
-- 
Bernardo Freitas Paulo da Costa

-- 
Esta mensagem foi verificada pelo sistema de antivírus e
 acredita-se estar livre de perigo.


=
Instruções para entrar na lista, sair da lista e usar a lista em
http://www.mat.puc-rio.br/~obmlistas/obm-l.html
=


Re: [obm-l] CN 2009

2013-09-04 Por tôpico Bernardo Freitas Paulo da Costa
2013/9/4 Hermann ilhadepaqu...@bol.com.br:
 From: Paulo Barclay Ribeiro
 To: obm-l@mat.puc-rio.br
 Sent: Wednesday, September 04, 2013 7:46 PM
 Subject: [obm-l] CN 2009

 Prezados, Boa noite.

 Peço uma orientação para resolver a seguinte questão do Colegio Naval de
 2009:

 Sobre o lado maior de um retangulode base 1 e altura 2 constroi-se um
 retangulo de base 2 e altura 3;sobre o maior lado desse último constroi-se
 um retangulo de base 3 e altura 4; e assim sucessivamente,até se construir o
 retangulo de base 99 e altura 100.Com quanto zeros termina o produto das
 areas de cada um desses retangulos?
 Desde já agradeço

 Um abraço
 paulo Barclay


 Refiz e deu 44 zeros, esqueci-me que o númeo era repetido, acho que agora
 acertei.

No PARI/GP, eu achei 46 zeros para 100! * 99!...



-- 
Bernardo Freitas Paulo da Costa

-- 
Esta mensagem foi verificada pelo sistema de antivírus e
 acredita-se estar livre de perigo.


=
Instruções para entrar na lista, sair da lista e usar a lista em
http://www.mat.puc-rio.br/~obmlistas/obm-l.html
=


Re: [obm-l] Projeto rumo ao ita

2013-09-04 Por tôpico Bernardo Freitas Paulo da Costa
2013/9/4 marcone augusto araújo borges marconeborge...@hotmail.com

 Sejam a,b,c,d números reais tais que a^2 + b^2 = c^2 + d^2 = 1 e
 ac + bd = 0.Determine o valor de ab + cd

 Eu pensei em vetores (a,b) e (c,d)
 O produto interno desses vetores,de acordo com o enunciado,é 0
 Então temos c = b e d = - a ou c = - b e d = a e em ambos os casos
 ocorre ab = - cd.Dai,segue que ab + cd = 0

 Daria pra resolver usando uma interpretação geométrica ou de outro modo?
Bom, eu fiz mais ou menos como você: os vetores u=(a,b) e v=(c,d) são
ortogonais e estão no círculo unitário. Uma versão levemente mais
geométrica diz que u=(cos x, sin x), v = (+- sin x, -+ cos x) para que
eles sejam ortogonais, logo o produto é sin(x) cos(x) para um, e -
sin(x)cos(x) no outro.

Outra idéia: ab = área de um triângulo com base num eixo e hipotenusa
da origem até u. Idem para cd. Só que triângulos nos quadrantes 2 e 4
têm o sinal trocado, a perpendicularidade dá a semelhança dos
triângulos, e a norma, a congruência das hipotenusas, logo dos
triângulos. Assim, |ab| = |cd| e elas têm sinal trocado.

Abraços,
-- 
Bernardo Freitas Paulo da Costa

-- 
Esta mensagem foi verificada pelo sistema de antivírus e
 acredita-se estar livre de perigo.


=
Instruções para entrar na lista, sair da lista e usar a lista em
http://www.mat.puc-rio.br/~obmlistas/obm-l.html
=


[obm-l] Re: [obm-l] Equação polinomial

2013-09-02 Por tôpico Bernardo Freitas Paulo da Costa
2013/9/2 marcone augusto araújo borges marconeborge...@hotmail.com

 Determine m sabendo que a equação x^4 - (3m+2)x^2 + m^2 = 0
 tem 4 raízes reais em progressão aritmética.

 Sejam - b, -a,a e b as raízes em PA.
 Devemos ter b = 3a (1) ;a+b = 3m+2 (2) e ab = m^2 (3)
 Resolvendo o sistema formado por (1) , (2) e (3) encontrei
 um valor bem feio pra m.
 Algo errado?

Você está usando Girard na equação quadrática em x^2. Cujas raízes são
a^2 e b^2, portanto as relações (2) e (3) estão erradas. E não esqueça
de verificar que quando você tiver terminado de resolver (1), (2) e
(3) para achar a^2 e b^2, que ambos sejam positivos !

-- 
Bernardo Freitas Paulo da Costa

-- 
Esta mensagem foi verificada pelo sistema de antivírus e
 acredita-se estar livre de perigo.


=
Instruções para entrar na lista, sair da lista e usar a lista em
http://www.mat.puc-rio.br/~obmlistas/obm-l.html
=


[obm-l] Re: [obm-l] Funções periódicas nos complexos

2013-09-02 Por tôpico Bernardo Freitas Paulo da Costa
2013/9/2 Artur Costa Steiner steinerar...@gmail.com:
 Olá amigos,
Oi Artur,

 Se f for uma função meromorfa, periódica e não constante, então f tem algo 
 análogo a um período fundamental, isto é, existe um período p cujo valor 
 absoluto é positivo e mínimo. Não sei se este p é único. Todos os múltiplos 
 inteiros de p são períodos e estão na reta definida pelo argumento de p. No 
 caso, por exemplo, de f(z) = exp(z), p = 2 pi i.

Não, este p não é único. Existem funções meromorfas cujos períodos são
1 e i, que são portanto mínimos e diferentes. Mas, fora este fenômeno
meio especial, o que você falou está perfeitamente certo. Para ser
formal: como f não é constante, existe um ponto z tal que f'(z) é
diferente de zero, e portanto uma vizinhança onde f(z) != f(w) para
todo w na vizinhança. Assim, o conjunto dos períodos {p complexo /
f(w+p) = f(w) para todo w} é um subgrupo aditivo de C, onde zero é
isolado. Assim (mesma demonstração que em R) existe um valor r = min =
inf { |p| / p período, p != 0 }.

 Mas pode haver outros períodos que não sejam múltiplos inteiros de p. É fácil 
 ver que estes últimos não estão sobre a reta acima citada. Ouvi, e não 
 consegui provar, que, dentre os períodos não múltiplos inteiros de p, há um 
 p' cuja distância à reta dos múltiplos inteiros de p é positiva e mínima. O 
 que eu consegui provar é que a distância euclidiana entre o conjunto dos 
 períodos da forma np, n inteiro, e o conjunto dos outros períodos é positiva.

É basicamente um argumento de inf = min em conjuntos discretos.
Considere todos os períodos que não estão na reta pZ, chame este
conjunto de PP. Eles estão todos a distância maior ou igual a r da
origem, e pela minimalidade de p, há apenas um número finito deles em
qualquer disco de raio R. Considere portanto uma aplicação f : PP
inter D_R x [0,p] - R dada pela distância de um ponto periódico em
D_R e um ponto no segmento 0-p. Ela é contínua, logo admite um mínimo
diferente de zero. Agora, se R é suficientemente grande, por conta da
simetria de translação, este mínimo será também o mínimo da função F :
PP x pR - R distância. (Formalize este último argumento. Dica: comece
estimando o mínimo com um ponto qualquer q em PP.)

Hum, relendo tudo aqui, eu vi que eu me confundi com a reta dos
múltiplos inteiros e provei que o mínimo é para todos os pontos da
reta, e não apenas (como fica claro na parte seguinte) que são apenas
os pontos pZ e todos os outros pontos que você está falando. A
demonstração, entretanto, é exatamente a mesma. Não dá pra fugir da
compacidade ;-).

Dê uma olhada em lattices na Wikipedia (em inglês, ou, com mais
figuras ainda, réseaux em francês). (adendo: palavrinha chata, ela
se diz reticulado ou retículo em português... muitas diferenças em
línguas simples!)

 Parece também que p/p' não é real. E que o conjunto de todos os períodos é o 
 conjunto das combinações lineares inteiras de p e de p'. Que tais combinações 
 são períodos é imediato, mas além disto não há nenhum período que não se 
 enquadre em tais combinações.

Isso é um argumento muito legal de álgebra linear com coeficientes
inteiros / racionais. A idéia intuitiva é que um reticulado com mais
do que n geradores L.I. sobre Q, todos os geradores em R^n, não é
discreto. Assim, se p/q fosse real, teríamos dois geradores
independentes sobre Q, logo uma seqüência de pontos z_n - 0 onde
f(z_n) = f(0), logo f seria constante (e aqui você usa que f é
analítica).

 Eu estou certo? Alguém conhece este assunto?

Se você quiser olhar para as funções meromorfas (bi-)periódicas, estas
são as belíssimas funções p de Weierstrass, e têm a ver com Teo dos
Números e geometria complexa. Se for mais a parte de Álgebra Linear,
tem também várias coisas (e também muitas coisas de Teo dos Números,
claro), e daí eu conheço menos...

 Abraços

 Artur

Abraços,
-- 
Bernardo Freitas Paulo da Costa

-- 
Esta mensagem foi verificada pelo sistema de antivírus e
 acredita-se estar livre de perigo.


=
Instruções para entrar na lista, sair da lista e usar a lista em
http://www.mat.puc-rio.br/~obmlistas/obm-l.html
=


[obm-l] Re: [obm-l] Funções periódicas nos complexos

2013-09-02 Por tôpico Bernardo Freitas Paulo da Costa
2013/9/2 Bernardo Freitas Paulo da Costa bernardo...@gmail.com:
 2013/9/2 Artur Costa Steiner steinerar...@gmail.com:
 Olá amigos,
 Oi Artur,

 Se f for uma função meromorfa, periódica e não constante, então f tem algo 
 análogo a um período fundamental, isto é, existe um período p cujo valor 
 absoluto é positivo e mínimo. Não sei se este p é único. Todos os múltiplos 
 inteiros de p são períodos e estão na reta definida pelo argumento de p. No 
 caso, por exemplo, de f(z) = exp(z), p = 2 pi i.

 Não, este p não é único. Existem funções meromorfas cujos períodos são
 1 e i, que são portanto mínimos e diferentes. Mas, fora este fenômeno
 meio especial, o que você falou está perfeitamente certo. Para ser
 formal: como f não é constante, existe um ponto z tal que f'(z) é
 diferente de zero, e portanto uma vizinhança onde f(z) != f(w) para
 todo w na vizinhança. Assim, o conjunto dos períodos {p complexo /
 f(w+p) = f(w) para todo w} é um subgrupo aditivo de C, onde zero é
 isolado. Assim (mesma demonstração que em R) existe um valor r = min =
 inf { |p| / p período, p != 0 }.

Ah, sim, faltou o exemplo:
considere a função

f(z) = sum_{m,n inteiros} 1/(z - m - n*i)^3

que é (por definição!) periódica de períodos 1 e i. É um pouco mais
chatinho ver que ela é meromorfa, porque daí você tem que provar que
ela é
- uma série convergente para z fora do reticulado {1,i} (use que 1/z^3
é integrável em R^2)
- que a derivada desta série convergente é também uma série
convergente, uniformemente sobre os compactos que não intersectam o
reticulado
porque daí ela será uma função com derivada contínua e z-linear,
portanto holomorfa em todos o C menos nos pólos do reticulado.
Para ver que ela é meromorfa nos pontos do reticulado, isole o termo
1/z^3 numa vizinhança de zero, repita os argumentos de cv uniforme e
veja que a derivada existe.
Como f é periódica, acabou.

Outra demonstração: tome |z|  1/3, expanda todos os termos exceto
1/z^3 em potências de z, usando 1/(z - a) = soma da série geométrica,
depois derivando a série 2 vezes, troque a ordem das somas (atenção
para aplicar Fubini direitinho) e veja que a série assim obtida é um
desenvolvimento de Laurent.

Essa é uma das funções de Weierstrass. Existe uma outra, mais
importante, que é a primitiva desta, mas é mais difícil mostrar que a
primitiva é periódica ;-) (e também é mais difícil mostrar que a
primitiva é uma função meromorfa bonitinha).

Abraços,
-- 
Bernardo Freitas Paulo da Costa

-- 
Esta mensagem foi verificada pelo sistema de antivírus e
 acredita-se estar livre de perigo.


=
Instruções para entrar na lista, sair da lista e usar a lista em
http://www.mat.puc-rio.br/~obmlistas/obm-l.html
=


Re: [obm-l] O Paradoxo da Flecha - Off Topic

2013-08-26 Por tôpico Bernardo Freitas Paulo da Costa
2013/8/26 Paulo Cesar pcesa...@gmail.com:
 Esse problema foi resolvido com o advento do Cálculo infinitesimal. Há
 muitas versões da mesma ideia, como o paradoxo de Aquiles.
O paradoxo da flecha e o de Aquiles e a Tartaruga foram resolvidos com
a construção dos números reais, também... para dar sentido completo às
somas de números cada vez menores.

Mas, da forma como enunciado, o paradoxo não é difícil de se refutar:

 . O paradoxo da flecha*

 Para um objeto se mover, sua posição no espaço deve mudar, certo? Pois
 bem, esse paradoxo do filósofo grego Zeno de Eleia (495 a.C 430 a.C) diz que
 os objetos não se movem. Considere um instante como uma fotografia, cada
 espaço de tempo é uma fotografia na qual o objeto está parado.

Já temos um problema aqui. Ao considerar que num instante, a flecha
está parada. Nada disso, ela TEM velocidade, mas observar o movimento
só faz sentido AO LONGO do tempo, não numa fotografia, e é por isso
que você não vê a flecha se mexer nesta dita fotografia.

Aliás, alguém tem uma referência original do Zenão (Zeno é como os
americanos traduzem Ζήνων) ? A gente sempre ouve falar em fotografia,
mas será que naquela época esta já era uma idéia comum?

Abraços,
-- 
Bernardo Freitas Paulo da Costa

-- 
Esta mensagem foi verificada pelo sistema de antiv�rus e
 acredita-se estar livre de perigo.


=
Instru��es para entrar na lista, sair da lista e usar a lista em
http://www.mat.puc-rio.br/~obmlistas/obm-l.html
=


Re: [obm-l] Problemas interessantes

2013-08-25 Por tôpico Bernardo Freitas Paulo da Costa
2013/8/25 Benedito bened...@ufrnet.br:
 Eduardo,

 A sua observação faz sentido. O que falta é a vírgula !!!:

 Um triângulo equilátero de lado 2012 está dividido em 2012  triângulos
 equiláteros menores, de lado 1.
Continua errado. As áreas não batem.

Eu acho que é divida o triângulo de lado 2012 em montes de
trianglinhos de lado 1 com lados paralelos aos originais. Só tem um
jeito de fazer isso. E o problema começa na frase seguinte.

Dica: fazer esta mesma operação (e botar as mesmas formigas, mas
menos) em triângulos de lado 1 e 2.

Abraços,
-- 
Bernardo Freitas Paulo da Costa

-- 
Esta mensagem foi verificada pelo sistema de antivírus e
 acredita-se estar livre de perigo.


=
Instruções para entrar na lista, sair da lista e usar a lista em
http://www.mat.puc-rio.br/~obmlistas/obm-l.html
=


Re: [obm-l] Como que faz??

2013-08-23 Por tôpico Bernardo Freitas Paulo da Costa
2013/8/23  douglas.olive...@grupoolimpo.com.br:
 Olá , alguns alunos do ensino médio da instituição onde trabalho me deram
 alguns problemas do site https://brilliant.org/

 PROBLEMA 1: Dada uma função f:R-R tal que f(2x^2 -1)=2(f(x))^2 -1 e f(x) é
 um polinômio de grau 13, sendo assim determine o coeficiente de x^5 de f(x).
Bom, f(x) é dada por 14 coeficientes a_i. A equação que você tem dá um
monte de condições sobre estes coeficientes: para cada x, tem uma
condição.

Por exemplo, x = 0 dá f(-1) = 2f(0)^2 - 1, f(-1) é uma soma alternada,
f(0)^2 é apenas (a_0)^2. Escolhendo um monte de x's, você obtém
bastante equações, e resolve.

Dá pra fazer (um pouco) menos grotesco, porque você pode escrever (a_0
+ a_1 * x + a_2 * x^2 +  + a_13 * x^13)^2, separando por grau.
Duas funções polinomiais em R são iguais se e somente se os
coeficientes forem iguais. Assim, identifique os coeficientes de ambos
os lados, e parta pro abraço.

Pra entender porque isso sempre dá certo, vale a pena lembrar de
Álgebra Linear (também se ensina um pouco desse tipo de intuição em
cursos de Geometria Algébrica classica).

Abraços
-- 
Bernardo Freitas Paulo da Costa

-- 
Esta mensagem foi verificada pelo sistema de antivírus e
 acredita-se estar livre de perigo.


=
Instruções para entrar na lista, sair da lista e usar a lista em
http://www.mat.puc-rio.br/~obmlistas/obm-l.html
=


<    1   2   3   4   5   6   7   >